Формула сопротивления при параллельном соединении резисторов: Онлайн-калькулятор расчета последовательного и параллельного соединения резисторов

Содержание

Формула расчета сопротивления при параллельном соединении резистора

Параллельное соединение резисторов — одно из двух видов электрических соединений, когда оба вывода одного резистора соединены с соответствующими выводами другого резистора или резисторов. Зачастую резисторы соединяют последовательно или параллельно для того, чтобы создать более сложные электронные схемы.

Схема параллельного соединения резисторов показан на рисунке ниже. При параллельном соединении резисторов, напряжение на всех резисторах будет одинаковым, а протекающий через них ток будет пропорционален их сопротивлению:

Формула параллельного соединения резисторов

Общее сопротивление нескольких резисторов соединенных параллельно определяется по следующей формуле:

Ток, протекающий через отдельно взятый резистор, согласно закону Ома, можно найти по формуле:

При разработке устройства, возникла необходимость установить резистор с сопротивлением 8 Ом. Если мы просмотрим весь номинальный ряд стандартных значений резисторов, то мы увидим, что резистора с сопротивлением в 8 Ом в нем нет.

Выходом из данной ситуации будет использование двух параллельно соединенных резисторов. Эквивалентное значение сопротивления для двух резисторов соединенных параллельно рассчитывается следующим образом:

Данное уравнение показывает, что если R1 равен R2, то сопротивление R составляет половину сопротивления одного из двух резисторов. При R = 8 Ом, R1 и R2 должны, следовательно, иметь значение 2 × 8 = 16 Ом.
Теперь проведем проверку, рассчитав общее сопротивление двух резисторов:

Таким образом, мы получили необходимое сопротивление 8 Ом, соединив параллельно два резистора по 16 Ом.

Пример расчета №2

Найти общее сопротивление  R из трех параллельно соединенных резисторов:

Общее сопротивление R рассчитывается по формуле:

Этот метод расчета может быть использованы для расчета любого количества отдельных сопротивлений соединенных параллельно.

Один важный момент, который необходимо запомнить при расчете параллельно соединенных резисторов – это то, что общее сопротивление всегда будет меньше, чем значение наименьшего сопротивления в этой комбинации.

Как рассчитать сложные схемы соединения резисторов

Более сложные соединения резисторов могут быть рассчитаны путем систематической группировки резисторов. На рисунке ниже необходимо посчитать общее сопротивление цепи, состоящей из трех резисторов:

Резисторы R2 и R3 соединены последовательно (группа 2). Они в свою очередь соединены параллельно с резистором R1 (группа 1).

Последовательное соединение резисторов группы 2 вычисляется как сумма сопротивлений R2 и R3:

В результате мы упрощаем схему в виде двух параллельных резисторов. Теперь общее сопротивление всей схемы можно посчитать следующим образом:

Расчет более сложных соединений резисторов можно выполнить используя законы Кирхгофа.

Ток, протекающий в цепи параллельно соединенных резисторах

Общий ток I протекающий в цепи параллельных резисторов равняется сумме отдельных токов, протекающих во всех параллельных ветвях, причем ток в отдельно взятой ветви не обязательно должен быть равен току в соседних ветвях.

Несмотря на параллельное соединение, к каждому резистору приложено одно и то же напряжение. А поскольку величина сопротивлений в параллельной цепи может быть разной, то и величина протекающего тока через каждый резистор тоже будет отличаться (по определению закона Ома).

Рассмотрим это на примере двух параллельно соединенных резисторов. Ток, который течет через каждый из резисторов ( I1 и I2 ) будет отличаться друг от друга поскольку сопротивления резисторов R1 и R2 не равны.
Однако мы знаем, что ток, который поступает в цепь в точке «А» должен выйти из цепи в точке «B» .

Первое правило Кирхгофа гласит: «Общий ток, выходящий из цепи равен току входящий в цепь».

  • Таким образом, протекающий общий ток в цепи  можно определить как:
  • I = I1 + I2
  • Затем с помощью закона Ома можно вычислить ток, который протекает через каждый резистор:
  • Ток, протекающий в R1 = U ÷ R1 = 12 ÷ 22 кОм = 0,545 мА
  • Ток, протекающий в R 2 = U ÷ R2 = 12 ÷ 47 кОм = 0,255 мА
  • Таким образом, общий ток будет равен:
  • I = 0,545 мА + 0,255 мА = 0,8 мА
  • Это также можно проверить, используя закон Ома:
  • I = U ÷ R = 12 В ÷ 15 кОм = 0,8 мА (то же самое)
  • где 15кОм — это общее сопротивление двух параллельно соединенных резисторов (22 кОм и 47 кОм)
  • И в завершении хочется отметить, что большинство современных резисторов маркируются цветными полосками и назначение ее можно узнать здесь.

Параллельное соединение резисторов — онлайн калькулятор

Чтобы быстро вычислить общее сопротивление двух и более резисторов, соединенных параллельно, вы можете воспользоваться следующим онлайн калькулятором:

Подведем итог

Когда два или более резистора соединены так, что оба вывода одного резистора соединены с соответствующими выводами другого резистора или резисторов, то говорят, что они соединены между собой параллельно. Напряжение на каждом резисторе внутри параллельной комбинации одинаковое, но токи, протекающие через них, могут отличаться друг от друга, в зависимости от величины сопротивлений каждого резистора.

Эквивалентное или полное сопротивление параллельной комбинации всегда будет меньше минимального сопротивления резистора, входящего в параллельное соединение.

Источник: http://www.joyta.ru/7362-parallelnoe-soedinenie-rezistorov/

Последовательное и параллельное соединение резисторов

Последовательное соединение – это соединение двух или более резисторов в форме цепи, в которой каждый отдельный резистор соединяется с другим отдельным резистором только в одной точке.

Общее сопротивление Rобщ

При таком соединении, через все резисторы проходит один и тот же электрический ток. Чем больше элементов на данном участке электрической цепи, тем «труднее» току протекать через него. Следовательно, при последовательном соединении резисторов их общее сопротивление увеличивается, и оно равно сумме всех сопротивлений.

Напряжение при последовательном соединении

Напряжение при последовательном соединении распределяется на каждый резистор согласно закону Ома:

Т.е чем большее сопротивление резистора, тем большее напряжение на него падает.

Параллельное соединение резисторов

Параллельное соединение – это соединение, при котором резисторы соединяются между собой обоими контактами. В результате к одной точке (электрическому узлу) может быть присоединено несколько резисторов.

Общее сопротивление Rобщ

При таком соединении, через каждый резистор потечет отдельный ток. Сила данного тока будет обратно пропорциональна сопротивлению резистора. В результате общая проводимость такого участка электрической цепи увеличивается, а общее сопротивление в свою очередь уменьшается.

Таким образом, при параллельном подсоединении резисторов с разным сопротивлением, общее сопротивление будет всегда меньше значения самого маленького отдельного резистора.

Формула общей проводимости при параллельном соединении резисторов:

Формула эквивалентного общего сопротивления при параллельном соединении резисторов:

Для двух одинаковых резисторов общее сопротивление будет равно половине одного отдельного резистора:

Соответственно, для n одинаковых резисторов общее сопротивление будет равно значению одного резистора, разделенного на n.

Напряжение при параллельном соединении

Напряжение между точками A и B является как общим напряжением для всего участка цепи, так и напряжением, падающим на каждый резистор в отдельности. Поэтому при параллельном соединении на все резисторы упадет одинаковое напряжение.

Электрический ток при параллельном соединении

Через каждый резистор течет ток, сила которого обратно пропорциональна сопротивлению резистора. Для того чтобы узнать какой ток течет через определенный резистор, можно воспользоваться законом Ома:

Смешанное соединение резисторов

Смешанным соединением называют участок цепи, где часть резисторов соединяются между собой последовательно, а часть параллельно. В свою очередь, смешанное соединение бывает последовательного и параллельного типов.

Общее сопротивление Rобщ

Для того чтобы посчитать общее сопротивление смешанного соединения:

  • Цепь разбивают на участки с только пареллельным или только последовательным соединением.
  • Вычисляют общее сопротивление для каждого отдельного участка.
  • Вычисляют общее сопротивление для всей цепи смешанного соединения.

Так это будет выглядеть для схемы 1:

Также существует более быстрый способ расчета общего сопротивления для смешанного соединения. Можно, в соответствии схеме, сразу записывать формулу следующим образом:

  • Если резисторы соединяются последоватеьно — складывать.
  • Если резисторы соединяются параллельно — использовать условное обозначение «||».
  • Подставлять формулу для параллельного соединения где стоит символ «||».

Так это будет выглядеть для схемы 1:

После подстановки формулы параллельного соединения вместо «||»:

Источник: http://hightolow.ru/resistor3.php

Параллельное соединение сопротивлений в электрической цепи. Параллельное соединение конденсаторов и катушек

Параллельное соединение электрических элементов (проводников, сопротивлений, емкостей, индуктивностей) — это такое соединение, при котором подключенные элементы цепи имеют два общих узла подключения.

Другое определение: сопротивления подключены параллельно, если они подключены одно и той же паре узлов. 

Графическое обозначение схемы параллельного соеднинения

На приведенном рисунке показана схема параллельное подключения сопротивлений R1, R2, R3, R4. Из схемы видно, что все эти четыре сопротивления имеют две общие точки (узла подключения). 

В электротехнике принято, но не строго требуется, рисовать провода горизонтально и вертикально. Поэтому эту же схему можно изобразить, как на рисунке ниже. Это тоже параллельное соединение тех же самых сопротивлений.

Формула для расчета параллельного соединения сопротивлений

При параллельном соединении обратная величина от эквивалентного сопротивления равна сумме обратных величин всех параллельно подключенных сопротивлений. Эквивалентная проводимость равна сумме всех параллельно подключенных проводимостей электрической схемы.

Для приведенной выше схемы эквивалентное сопротивление можно рассчитать по формуле:

В частном случае при подключении параллельно двух сопротивлений:

Эквивалентное сопротивление цепи определяется по формуле:

 В случае подключения «n» одинаковых сопротивлений, эквивалентное сопротивление можно рассчитать по частной формуле:

Формулы для частного рассчета вытекают из основной формулы. 

Формула для расчета параллельного соединения емкостей (конденсаторов)

При параллельном подключении емкостей (конденсаторов) эквивалентная емкость равна сумме параллельно подключенных емкостей:

 

Формула для расчета параллельного соединения индуктивностей

  • При параллельном подключении индуктивностей, эквивалентная индуктивность рассчитывается так же, как и эквивалентное сопротивление при параллельном соединении: 
  •  
  • Необходимо обратить внимание, что в формуле не учтены взаимные индуктивности.

Пример свертывания параллельного сопротивления  

Для участка электрической цепи необходимо найти параллельное соединение сопротивлений выполнить их преобразование до одного.

Из схемы видно, что параллельно подключены только R2 и R4. R3 не параллельно, т.к. одним концом оно подключено к источнику ЭДС E1. R1 — одним концом подключено к R5, а не к узлу. R5 — одним концом подключено к R1, а не к узлу. Можно так же говорить, что последовательное соединение сопротивлений R1 и R5 подключено параллельно с R2 и R4.

Рассчитать эквивалентное сопротивлений R14 можно по формуле для двух сопротивлений.

Ток при параллельном соединении

При параллельном соединении сопротивлений ток через каждое сопротивление в общем случае разный. Величина тока обратно пропорциональна величине сопротивления.

Напряжение при параллельном соединении 

При параллельном соединении разность потенциалов между узлами, объединяющими элементы цепи, одинакова для всех элементов.

Применение параллельного соединения

1. В промышленности изготавливаются сопротивления определенных величин. Иногда необходимо получить значение сопротивления вне данных рядов. Для этого можно подключить несколько сопротивлений параллельно. Эквивалентное сопротивление всегда будет меньше самого большого номинала сопротивления.

2. Делитель токов.

Источник: https://kurstoe.ru/osnovnie-svedeniya/preobrazovanie-tcepej/parallelnoe-soedinenie.html

Последовательное и параллельное соединение резисторов

Последовательное и параллельное соединение резисторов в схемах являются самыми распространенными, также — это база для расчета более сложных схем.

Последовательное подключение

Начнем с последовательного соединения. По этой схеме каждый резистор подключается с другим только в одной точке, их может быть в цепи 2, 3 и больше.

Рис. Последовательное подключение.

Обозначение:

Обозначим сопротивления: R1, R2, R3 и напряжение источника в цепи Uц. При подключении источника питания в ней начнет протекать ток Iц. В цепи с последовательным соединением ток протекает по всем резисторам один за другим.

Поскольку ток течет через все резисторы их сопротивления и ток суммируется, Iц = I1+I2+I3, Rц = R1 +R2 + R3, чем больше отдельно взятое сопротивление, тем тяжелее электронам преодолевать участок цепи. Мощность резисторов при последовательном и параллельном соединении рассчитывается по разным формулам.

В последовательных цепях — складываем, в параллельных — это обратно пропорциональная величина.

Параллельное соединение

Рис. Параллельное подключение.

Данный вид подключения характерен тем, что все элементы цепи соединяется выводами в одной точке друг другу, т.е. точка входа и выхода всех нагрузок сходятся в одну точку (или еще одно обозначение на схемах — //). Электроток, двигаясь по проводнику, дойдя до общего соединения делится на количество имеющихся веток. Если представить движение воды в трубе, то можно сказать, что вода двигающиеся по одной трубе, равномерно перетекает в несколько отводов, подсоединенных к ней. В нашем случае заряженные электроны, двигающиеся по проводнику, также растекаются на количества предложенных веток в узле. Более наглядно это можно представить в виде формул: 1. Каждый вид соединения находится под одинаковым напряжением: U = U1 = U2; 2. Суммарная сила тока равняется суммарному значению тока каждого участка I = I1 + I2; 3. Сопротивление цепи равно сумме величина обратных сопротивлению участка: 1/R = 1/R1 + 17R2 + . . . + 1/Rn; 4. Сила тока пропорциональна сопротивлению каждого участка I1/I2=R2/R1.

Далее рассмотрим схему как работает не только последовательное параллельное, но и смешанное соединение резисторов.

Смешанное подключение

Рис. Смешанное подключение резисторов

В электрических схемах используются не только типовые схемы, но и смешанное, созданное из критерий определенных требований. Чаще всего в схемах встречается третий вариант, представляющий набор из элементарных типов схем. В смешанных участках учитываются не только элементы, но и направления движения тока. При вычислении мощности резисторов смешанного подключения используются формулы для параллельного и последовательного соединения резисторов, формула также является составной.

Основные законы электротехники, наиболее часто используемые для расчетов

Рассмотрим основные законы электротехники и свойства последовательного и параллельного соединения резисторов для участка цепи

Закон Ома

Напряжение находится по закону Ома по формуле I=U/R — чем больше сопротивление, тем меньше ток. Напряжение можно найти из этой же формулы. U=R*I, ток умножается на сопротивление. Запишем эту формулу для каждого участка U1=R1· I1, Un=Rn · In.

Законы Кирхгофа

Первый закон

Ещё один очень важный закон — это закон Кирхгофа. Для участка цепи постоянного тока их два.

Рис. иллюстрация к пояснению действия первого закона Кирхгофа.

Первый закон имеет формулировку: Сумма всех токов, входящих в узел и выходящих из него равна нулю. Если посмотреть на схему, I1 — это ток, который заходит в узел, I2 и I3 — это электроны, которые вытекают из него. Применяя формулировку первого закона можно записать формулу по-другому:

I1-I2+I3=0. В этой формуле знаки плюс имеют значения, которые прибывают в узел, минус, который отходит от него.

Второй закон Кирхгофа

Рис. иллюстрация к пояснению действия второго закона Кирхгофа.

Если к цепи с включенными сопротивлениями подключен один источник ЭДС (батарея питания) тогда всё понятно, можно обойтись законом Ома. А, если, источников несколько и схема с различным схемным расположением элементов, тогда вступает в силу второй закон, который гласит: сумма токов всех источников питания для замкнутого контура, равна сумме падений напряжения на всех сопротивлениях участка в этом контуре. E1- Е2 = — UR1 — UR2 или E1 = Е2 — UR1 — UR2.

Параллельное и последовательное соединение резисторов,  решение задач

Алгоритм расчёта смешанных подключений находится в тех же правилах, что и в элементарных схемах расчета последовательного и параллельного соединения резисторов. Ничего нового нет: нужно правильно разбить предложенную схему на пригодные для расчета участки. Участки, с элементами, подключены поочередно либо параллельно.

Рис. Порядок замещения при расчете сложных позиций более простыми.

Для решения задачи на последовательное и параллельное соединение резисторов необходимо правильно оценить цепи элементов. Рассмотрим схему №1 на рис.

На схеме присутствует параллельная и последовательная часть соединения элементов. Для расчета очень важно аккуратно, шаг за шагом упрощать цепи и не брать сразу всю схему (рис.1).

Как же правильно определить параллельное и последовательное соединение резисторов?

Для примера расчета возьмем резисторы R3, R4, которые подключены параллельно. Эквивалентный резистор этих элементов, будет равенRэ. = 1/R34 =1/R3 + 1/R4, после преобразования формулы и приведения к одному знаменателю получим R34 = R3 · R4 / (R3 + R4). Э. = 1/3+1/4 /(3+4) =1,7 Ом.

Далее видно, что приведённая эквивалентное R эк и R6 соединены последовательно, чтобы узнать сопротивление их необходимо сложить, тогда общее сопротивление будет равно R346 = R34 + R6, тогда Rэк346 = 1,7 + 6 = 7, 7 Ом. Заменяем на схеме одним общим элементом, теперь, позиция упрощается еще больше (рис 3).

Теперь образовалась ситуация — включение трех элементов в //. Как вычисляется такое соединение нам уже известно, 1/ R23465 = 1/ R2 +1/R346 + 1/R5 после вычисления правой части получаем 0,82 Ом. После окончательного вычисления получаем R23465 = 2,1 Ом. Здесь следует обратить внимание, что общее сопротивление получилось меньше самого меньшего из трех.

Заменяем эти сопротивление одним эквивалентным R23465. В конечном итоге все выглядит уже намного проще. Rц = Rэк + R1+ R2. R об. = R ц = 1,21 +7+1 =9,21 Ом. Из приведенного алгоритма расчёта видно, как из сложной схемы путем простого математического вычисления и применения правил сокращения резисторов участок становится простой и понятной.

Схема с подключением сопротивлений «треугольником»

Рис. Расчетная схема соединения резисторов в треугольник.

Иногда некоторые затруднения возникают при разборе схемы соединения в треугольник.

Рассмотрим на примере рисунка расчет резисторов по этому подключению. Из схемы видно, что R1 и R2 соединены последовательно Rэ12 будет соединяться R3 последовательно.

Затем Rэ123 соединяется с сопротивлением R4, R5 в последовательную цепь. Затем все это объединяется с Rэ в //.

Проведем несложные вычисления учитывая, что R1, R2, R4, R5 равняется 1 Ом. R3, R7 — 2 Ом.

RЭ1,2 = R1+R2 = 1+1=2 Ом.

Вычисляем параллельное подключение: Rэ 12 с R3. Rэ1,3 = (Rэ12*R3) /(Rэ12+R3) = (2*2) /(2+2) = 1Ом.

Далее мы видим последовательное: RЭ123 + R4 + R5 = 1+1+1 = 3 Ом. И последнее — Rэ123 4 5 с R6 — параллельное.

Общее сопротивление цепи Rц = Rоб = (RЭ1,2,3,4,5 *R6) /(RЭ1,2,3,4,5+R6) = (3 * 2) / (3+2) = 1,2 Ом. Как видно, что расчет подобного варианта также не сложный.

Расчет последовательного и параллельного подключения резисторов онлайн

Подсчитать значение мощность и сопротивлений подставляя их в формулы можно только в учебных целях, или, когда объемы не очень большие.

Наиболее практичный вариант расчета является онлайн калькуляторы, которые расположены на многочисленных интернет ресурсах.

Для расчёта любой сложности нужно правильно определить тип соединения резисторов последовательное или параллельное и внести данные для расчета в поля калькулятора.

Также такая форма расчета подойдет и для проверки результатов решения учебных задач.

Последовательное и параллельное соединение резисторов и конденсаторов

Электрические цепи состоят не только из резисторов, в них применяется большое количество различных деталей, например, конденсатор, которые подключаются в последовательное, // и смешанное соединение.

Рис. Замещения последовательно включенных элементов.

Определение этому элементу можно дать следующее: Конденсатор — это совокупность проводящих тел служащий для накопления электрического заряда. Элементарный конденсатор имеет две пластины, форма этих пластин может быть различной: сферической, круглой, цилиндрической, прямоугольной — по форме пластин разделяется и тип конденсатора.

Важное свойство. Одно из важных свойств конденсатора: если заряжается одна пластина конденсатора, то благодаря явлению электростатической индукции заряжается и вторая половина, но с противоположным знаком.

Устройство конденсатора

Плоский конденсатор состоит из двух плоских пластин отстоящих друг от друга на маленькое расстояние. У конденсатора к двум пластинам припаивается вывод всего их получается два.

Типовые схемы подключения конденсаторов

Рассмотрим различные виды подключения конденсатора.

Последовательное

Первый вид — это последовательное соединение. Предположим, что емкость этих конденсаторов будут равны.

Тогда заряды также будут равны: q1=q2=q3, как и в примере с резисторами,  сложный тип позиций с конденсатором можно упростить, заменив несколько элементов одним.

У элементов соединенных друг за другом, общая емкость будет обратно пропорциональная всем имеющимся элементам. То есть: Rэк будет равняться 1/С1 + 1/С2 +…. 1/Сn/

Напряжение складывается,  U эк = U1 + U2+ … Un.

Параллельное

Второй тип подключения конденсаторов — это соединение в паралель

Рис. Схема замещения элементов, включенных в параллель.

  • Соответственно эти конденсаторов обозначены C1, C2, … Cn заряды: Q1, Q2, … Qn и напряжение: U1, U2, … Un.
  • У элементов в // емкость складывается Сэ = C1 + C2 + … C n. Напряжение Un на каждом конденсаторе будет равно напряжению на эквивалентном
  • Uэ = U1 = U2 =… = Un — это особенность параллельного подсоединения всех элементов цепи.
  • Емкость будет складываться из суммы отдельных элементов Сэ =С1 + С2 + … Сп.

Рис. Расчетные позиции элементов при различном включении.

Простая позиция, которая не требует преобразования №1 — последовательное подключение. По известной формуле для этих поз. запишем 1/Сэ = 1/С1 +1/С2 +1/С3,  подставив формулу значения, которые даны в условии задачи, получим 1/Сэ = 1/С1 +1/С2 +1/С3 = 59 мФ.

Не требует преобразования и 2 схема: емкость общего конденсатора будет равняться сумме конденсаторов которые включены в параллельной цепи: Сэ =С1 +С2 +С3 Сэ = 100 + 200 + 500 = 800 мФ.

Рассмотрев рис. №3 видно, что пара конденсаторов включена параллельно и один последовательно. Алгоритм преобразования таких цепей мы уже рассматривали, поэтому: сразу же находим емкость конденсатора Сэ соединения: Сэ = С1+С2 = 200+500 = 700 мФ.

Теперь находим общие эквивалентную емкость элементов с последовательным подключением 1/Сэ = 1/С2,3 +1/ С1 = 89 мф. Практическая задача решена.

Источник: http://themechanic.ru/posledovatelnoe-i-parallelnoe-soedinenie-rezistorov/

Соединение резисторов

Радиоэлектроника для начинающих

О том, как соединять конденсаторы и рассчитывать их общую ёмкость уже рассказывалось на страницах сайта. А как соединять резисторы и посчитать их общее сопротивление? Именно об этом и будет рассказано в этой статье.

Резисторы есть в любой электронной схеме, причём их номинальное сопротивление может отличаться не в 2 – 3 раза, а в десятки и сотни раз. Так в схеме можно найти резистор на 1 Ом, и тут же неподалёку на 1000 Ом (1 кОм)!

Поэтому при сборке схемы либо ремонте электронного прибора может потребоваться резистор с определённым номинальным сопротивлением, а под рукой такого нет. В результате быстро найти подходящий резистор с нужным номиналом не всегда удаётся. Это обстоятельство тормозит процесс сборки схемы или ремонта. Выходом из такой ситуации может быть применение составного резистора.

Для того чтобы собрать составной резистор нужно соединить несколько резисторов параллельно или последовательно и тем самым получить нужное нам номинальное сопротивление. На практике это пригождается постоянно.

Знания о правильном соединении резисторов и расчёте их общего сопротивления выручают и ремонтников, восстанавливающих неисправную электронику, и радиолюбителей, занятых сборкой своего электронного устройства.

Последовательное соединение резисторов

  • В жизни последовательное соединение резисторов имеет вид:
  • Последовательно соединённые резисторы серии МЛТ
  • Принципиальная схема последовательного соединения выглядит так:

На схеме видно, что мы заменяем один резистор на несколько, общее сопротивление которых равно тому, который нам необходим.

Подсчитать общее сопротивление при последовательном соединении очень просто. Нужно сложить все номинальные сопротивления резисторов входящих в эту цепь. Взгляните на формулу.

  1. Общее номинальное сопротивление составного резистора обозначено как Rобщ.
  2. Номинальные сопротивления резисторов включённых в цепь обозначаются как R1, R2, R3,…RN.
  3. Применяя последовательное соединение, стоит помнить одно простое правило:

Из всех резисторов, соединённых последовательно главную роль играет тот, у которого самое большое сопротивление. Именно он в значительной степени влияет на общее сопротивление.

Что это значит?

Так, например, если мы соединяем три резистора, номинал которых равен 1, 10 и 100 Ом, то в результате мы получим составной на 111 Ом.

Если убрать резистор на 100 Ом, то общее сопротивление цепочки резко уменьшиться до 11 Ом! А если убрать, к примеру, резистор на 10 Ом, то сопротивление будет уже 101 Ом.

Как видим, резисторы с малыми сопротивлениями в последовательной цепи практически не влияют на общее сопротивление.

Параллельное соединение резисторов

  • Можно соединять резисторы и параллельно:
  • Два резистора МЛТ-2, соединённых параллельно
  • Принципиальная схема параллельного соединения выглядит следующим образом:

Для того чтобы подсчитать общее сопротивление нескольких параллельно соединённых резисторов понадобиться знание формулы. Выглядит она вот так:

Эту формулу можно существенно упростить, если применять только два резистора. В таком случае формула примет вид:

Есть несколько простых правил, позволяющих без предварительного расчёта узнать, каково должно быть сопротивление двух резисторов, чтобы при их параллельном соединении получить то, которое требуется.

Если параллельно соединены два резистора с одинаковым сопротивлением, то общее сопротивление этих резисторов будет ровно в два раза меньше, чем сопротивление каждого из резисторов, входящих в эту цепочку.

Это правило исходит из простой формулы для расчёта общего сопротивления параллельной цепи, состоящей из резисторов одного номинала. Она очень проста. Нужно разделить номинальное сопротивление одного из резисторов на общее их количество:

Здесь R1 – номинальное сопротивление резистора. N – количество резисторов с одинаковым номинальным сопротивлением.

Ознакомившись с приведёнными формулами, вы скажите, что все они справедливы для расчёта ёмкости параллельно и последовательно соединённых конденсаторов. Да, только в отношении конденсаторов всё действует с точностью до «наоборот”. Узнать подробнее о соединении конденсаторов можно здесь.

Проверим справедливость показанных здесь формул на простом эксперименте.

Возьмём два резистора МЛТ-2 на 3 и 47 Ом и соединим их последовательно. Затем измерим общее сопротивление получившейся цепи цифровым мультиметром. Как видим оно равно сумме сопротивлений резисторов, входящих в эту цепочку.

  1. Замер общего сопротивления при последовательном соединении
  2. Теперь соединим наши резисторы параллельно и замерим их общее сопротивление.
  3. Измерение сопротивления при параллельном соединении
  4. Как видим, результирующее сопротивление (2,9 Ом) меньше самого меньшего (3 Ом), входящего в цепочку. Отсюда вытекает ещё одно известное правило, которое можно применять на практике:

При параллельном соединении резисторов общее сопротивление цепи будет меньше наименьшего сопротивления, входящего в эту цепь.

Что ещё нужно учитывать при соединении резисторов?

Во-первых, обязательно учитывается их номинальная мощность. Например, нам нужно подобрать замену резистору на 100 Ом и мощностью 1 Вт. Возьмём два резистора по 50 Ом каждый и соединим их последовательно. На какую мощность рассеяния должны быть рассчитаны эти два резистора?

Поскольку через последовательно соединённые резисторы течёт один и тот же постоянный ток (допустим 0,1 А), а сопротивление каждого из них равно 50 Ом, тогда мощность рассеивания каждого из них должна быть не менее 0,5 Вт. В результате на каждом из них выделится по 0,5 Вт мощности. В сумме это и будет тот самый 1 Вт.

Данный пример достаточно грубоват. Поэтому, если есть сомнения, стоит брать резисторы с запасом по мощности.

Подробнее о мощности рассеивания резистора читайте тут.

Во-вторых, при соединении стоит использовать однотипные резисторы, например, серии МЛТ. Конечно, нет ничего плохого в том, чтобы брать разные. Это лишь рекомендация.

Главная » Радиоэлектроника для начинающих » Текущая страница

Также Вам будет интересно узнать:

Источник: https://go-radio.ru/connection-of-resistors.html

Параллельное соединение резисторов

Господа, в прошлый раз мы с вами говорили про последовательное сопротивление резисторов. Сегодня я бы хотел вам рассказать про другой возможный вид соединения – параллельное.

Чем различается последовательное и параллельное соединение я уже писал в предыдущей статье.  Но все-таки вытащу сюда картинку из той прошлой статьи, я ж знаю, что вам будет лень ходить по ссылкам .

  • А) – Последовательное соединение
  • В) – Параллельное соединение
  • Рисунок 1 – Последовательное и параллельное соединение
  • Как мы видим из рисунка 1, параллельное соединение – это такое соединение, при котором одни концы всех резисторов соединены в один узел, а другие концы – в другой узел.

Сейчас наша задача будет разобраться, как ведут себя токи, напряжения, сопротивления и мощности при таком подключении. Для этого прошу вас взглянуть на рисунок 2, где подробно разрисован расклад дел для параллельного соединения. Будем полагать, что мы знаем величины R1, R2 и R3, а также величину приложенного к схеме напряжения U. Про токи же мы ничего не знаем.

Рисунок 2 – Параллельное соединения

Что мы видим на рисунке 2? Ну, в первую очередь – два узла А и B. В узел А сходятся одни концы всех резисторов, а в узел В – другие концы. Пусть узел А имеет потенциал φ1, а узел В – потенциал φ2. Из рисунка 2 видно, что для всех резисторов R1, R2 и R3 у нас одна и та же разность потенциалов U.

Как следует из статьи про потенциалы, это означает, что напряжение на всех резисторах у нас одинаково и равно приложенному напряжению U. Это важный вывод, его следует хорошо запомнить.

С токами дело обстоит по-другому. Проанализируем рисунок 2 слева направо. Пусть у нас в цепи течет ток I. Течет он себе, течет, никого не трогает и тут вдруг натыкается на узел А. Что в этом случае говорит полюбившаяся вам статья про первый закон Кирхгофа? А то, что ток I в узле А разделится на три тока I1, I2, I3. При этом будет выполняться равенство

То есть через резистор R1 будет протекать ток I1, через резистор R2 – ток I2, а через резистор R3 – ток I3.

Итак, у нас в системе уже тихо-мирно текут себе три тока. И все хорошо, пока они не наткнуться на узел В. Тут снова вступает в силу первый закон Кирхгофа. Эти три тока I1, I2, I3 вновь соединятся в один ток I. Причем после узла В ток будет иметь такую же величину I, какой он был до узла А.

То есть если все вышесказанное воплотить в лаконичный язык наскальной живописи, положение дел можно представить себе вот так

Как же найти эти самые токи I1, I2, I3? Господа, полагаю, вы уже догадались, что на помощь нам придет горячо нами всеми любимый закон Ома. Действительно, мы знаем сопротивления резисторов и, кроме того, нам известно, что на всех них падает одно и тоже напряжение U. Поэтому легко находим токи

Отлично, мы разобрались с напряжениями и с токами в такой схеме. А помните в статье про последовательное сопротивление мы ловко преобразовали три резистора в один с эквивалентным им сопротивлением? Нельзя ли и здесь сделать что-то подобное? Оказывается, вполне себе можно. Как мы помним, токи в схеме распределены таким вот образом

  1. Обзовем эквивалентное сопротивление буковкой R. И подставим в это выражение только что найденные нами токи I1, I2, I3
  2. Видим, что здесь без проблем можно сократить левую и правую части на U. Получаем
  3. Господа, важный вывод: при параллельном соединении резисторов обратное эквивалентное сопротивление равно сумме обратных сопротивлений отдельных резисторов.
  4. То есть для упрощения различных расчетов электрических схем такую вот цепочку параллельно соединенных резисторов можно заменить одним резистором с соответствующим сопротивлением, как показано на рисунке 3.
  5. Рисунок 3 – Преобразование параллельного соединение

Весьма частый случай на практике, когда соединены параллельно не много резисторов, а всего два. Поэтому полезно знать наизусть итоговое сопротивление такой схемы. Давайте посмотрим, чему оно равно:

То есть, если у вас два сопротивления соединены параллельно, то по этой формуле вы легко высчитаете общее сопротивление. Рассмотрим пример. Пусть у нас параллельно соединены два резистора 10 кОм и 15 кОм. Чему равно их общее сопротивление?

Заметьте, господа, итоговое сопротивление у нас получилось 6 кОм, что меньше 10 кОм и 15 кОм. То есть при параллельном соединении общее сопротивление меньше любого из составляющих.

Это всегда верно для любого количества резисторов, а не только для двух. Итоговое сопротивление всегда уменьшается (в отличии от последовательного сопротивления, где итоговое сопротивление всегда растет).

Этот факт полезно запомнить.

Еще один часто встречающийся на практике случай – когда параллельно соединены несколько резисторов с одинаковым сопротивлением. Допустим, каждый из них обладает сопротивлением R1 и всего их N штук. Тогда по нашей общей формуле для эквивалентного сопротивления

  • То есть при параллельном соединении N одинаковых резисторов с сопротивлением R1 итоговое сопротивление будет в N раз меньше этого самого сопротивления R1.
  • Так-с, с током разобрались, с напряжением разобрались, с эквивалентным сопротивлением вроде тоже…осталась мощность. Для этого воспользуемся вот этим выражением, которое мы писали чуть выше в статье
  • Умножим левую и правую части на напряжение U.
  • Как мы помним из статьи про мощность произведение тока на напряжение есть мощность. То есть мы можем записать
  • где Р – мощность, выдаваемая источником;
  • P1 – мощность, рассеиваемая на резисторе R1;
  • P2 – мощность, рассеиваемая на резисторе R2;
  • P3 – мощность, рассеиваемая на резисторе R3.

Заметьте, господа, формула в точности такая же, как и для случая последовательного соединения резисторов. И там и там мощность, выдаваемая источником, равна сумме мощностей, рассеиваемых на резисторах цепи.

Итак, господа, мы рассмотрели основные соотношения при параллельном соединении резисторов. Теперь осталось поговорить, где это параллельное соединение можно использовать и для чего.

1) Ну, во-первых, параллельное соединение применяют во всех случаях, когда хотят запитать несколько нагрузок от одного источника напряжения. При этом пользуются тем свойством, что при параллельном соединении напряжения на всех нагрузках одинаково.

То есть, допустим, вы берете источник напряжения, выставляете на нем напряжение 5 В и цепляете к этому источнику сразу несколько своих устройств. Узлами А и В в этом случае будут клеммы источника. На каждое из устройств в этом случае придет напряжение 5 В.

Да и все устройства в вашей квартире (лампочки, компьютеры, телевизоры и все прочее) соединены между собой параллельно.

2) Второе возможное применение встречается не так часто, но, думаю, о нем тоже следует рассказать. Допустим, вы делаете какую-то схему, где необходим очень точный подгон сопротивления. Скажем, надо получить сопротивление 6 кОм. Такое сопротивление найти нелегко, их просто не продают. Зато у вас есть два сопротивления 10 кОм и 15 кОм.

Вы их соединяете параллельно и получаете требуемые 6 кОм. Как показывает практика, 3 параллельных резисторов достаточно для получения итогового результирующего сопротивления требуемого номинала с весьма хорошей точностью. Конечно, таких вещей лучше избегать и, если есть возможность, всегда стараться применять стандартные сопротивления.

Но бывают случаи, когда это невозможно, и тогда приходит на помощь этот метод.

3) Третий пункт будет немного похож на первый. Его суть заключается в следующим. Допустим, нам надо снять с источника питания 10 Вт мощности. А у нас в наличии только резисторы, которые позволяют рассеивать на себе 1 Вт. Что делать? Можно соединить 10 резисторов параллельно и с каждого снимать по 1 Вт. Мы же помним нашу формулу

Конечно, лучше брать не 10 резисторов, а хотя бы 15 и рассеивать на них меньше, чем 1 Вт. Работать на пределе никогда не следует.

Кстати, тут очень вовремя к моменту написания статьи пришли платы с производства! Господа, прошу вас взглянуть на рисунок 4.

Рисунок 4 – Плата нагревателя

На нем изображена плата нагревателя (флешка для масштаба). В чем суть? Имеется весьма сложное устройство, предназначенное для работы в арктических условиях.

Найти же компоненты, которые надежно функционировать при температурах минус 55 градусов и при этом стоят адекватных денег и обладают адекватными размерами бывает непросто. Обычно элементная база в лучшем случае рассчитана на минус 40 градусов.

И было принято решение разработать вот такой вот нагреватель для прогрева чувствительных к холоду аналоговых узлов устройства. Он управляется с микроконтроллера и автоматически включается при температурах меньше минус 40 градусов.

Как вы можете видеть из рисунка 4, этот нагреватель представляет собой 30 параллельно соединенных резисторов с сопротивлениями 150 Ом. Каждый резистор, согласно документации, способен рассеивать до 1 Вт мощности. Используя изученные формулки, мы можем посчитать, что в сумме такая система обладает сопротивлением

  1. и теоретически может рассеивать мощность

Ну, с сопротивлением вопросов нет, оно действительно равно 5 Ом. Ну, плюс-минус 5 % на допуск резисторов, что в данном случае вообще не критично. А вот с мощностью тут не так все однозначно. Помните про закон Джоуля-Ленца, который мы рассматривали? Резисторы будут греться, причем не слабо.

Как показывает практика, если нагружать резисторы по полной, то есть рассеивать на каждом по 1 Вт, то в течении нескольких секунд их температура улетит за 150 градусов. Такая высокая температура критична для резистора и может привести к его разрушению.

Я был готов к такому развитию событий, поэтому заложил для платы нагревателя максимальное напряжение 9 вольт. Это значит, что на каждом резисторе будет выделяться

  • что почти в два раза меньше максимально допустимой мощности в 1 Вт. В сумме на всей плате выделялось, соответственно

Эксперимент показал, что резисторы достигли температуры с комнатных 25 градусов до критичных 120 градусов приблизительно за 10 секунд работы и температура продолжала уверенно расти.

Очевидно, если оставить на длительное время включенным такой нагреватель при комнатной температуре, он неминуемо выйдет из строя.

Возможно, при работе на минус 55 градусах перегрев бы не был столь критичным, однако хотелось исключить вариант спалить плату на столе, поэтому я понизил напряжение, подаваемое на плату на 3 вольта: стал подавать 6 вольт. Теперь на каждом резисторе рассеивалось

  1. а на всей плате

Теперь температура поднималась до 100-110 градусов примерно за 30-40 секунд работы и оставалась на этом уровне (выходила в точку термодинамического равновесия). Эта температура вполне подходит для нагревателя.

Однако пока это были лишь эксперименты на столе при комнатной температуре, главный эксперимент – в термокамере на минус 55 градусах – впереди. Возможно, по его результатам потребуется чуть увеличить рассеиваемую мощность.

А может все останется как есть и этой мощности будет достаточно для вывода девайса на режим за адекватное время, время покажет .

На сегодня все, господа. Удачи вам и до новых встреч!

Источник: http://myelectronix.ru/postoyannyy-tok/40-parallelnoe-soedinenie-rezistorov

Как отличается параллельное и последовательное соединение резисторов?

Большое разнообразие схем основано на двух видах соединений – последовательное параллельное. Для каждого типа существуют свои собственные законы и принципы. Именно это и позволяет создавать устройства с самыми различными техническими параметрами, в том числе и резисторы. Что же такое резистор?

Резистор – радиодеталь, созданная для контроля напряжения и тока в цепи, увеличивая либо понижая его. Резисторы могут быть двух видов – постоянные и переменные. Так, например, светодиоды требуют для себя совсем небольшого тока. Для этого в электрическую цепочку перед светодиодом устанавливается резистор, который обеспечивает необходимое напряжение для работы последнего.

В статье подробны рассмотрены все аспекты последовательного и параллельного подключения резисторов. Бонусом к статье являются видеоролик и детальная информационная статья на рассматриваемую тему.

Последовательное подключение

Начнем с последовательного соединения. По этой схеме каждый резистор подключается с другим только в одной точке, их может быть в цепи 2, 3 и больше. Обозначим сопротивления: R1, R2, R3 и напряжение источника в цепи Uц. При подключении источника питания в ней начнет протекать ток Iц. В цепи с последовательным соединением ток протекает по всем резисторам один за другим.

Поскольку ток течет через все резисторы их сопротивления и ток суммируется, Iц = I1+I2+I3, Rц = R1 +R2 + R3, чем больше отдельно взятое сопротивление, тем тяжелее электронам преодолевать участок цепи. Мощность резисторов при последовательном и параллельном соединении рассчитывается по разным формулам. В последовательных цепях — складываем, в параллельных — это обратно пропорциональная величина.

Последовательное соединение характеризуется тем, что элементы идут друг за другом. Конец одного подключается к началу другого. При подключении полученной цепочки к источнику тока получается кольцо.

Теоретическая часть

Последовательное соединение характерно тем, что через все элементы протекает ток одинаковой силы. То есть, если цепочка состоит из двух резисторов R1 и R2 (как на рисунке ниже), то ток протекающий через каждое из них и любую другую часть цепи будет одинаковой (I = I1 = I2).

Суммарное сопротивление всей цепи последовательно соединенных резисторов считается как сумма сопротивлений всех ее элементов. То есть, номиналы складывают. R = R1 + R2 — это и есть формула расчета сопротивления при последовательном соединении резисторов. Если элементов больше двух, будет просто больше слагаемых.

Еще одно свойство последовательного соединения — на каждом элементе напряжение отличается. Ток в цепи одинаковый, а напряжение на резисторе зависит от его номинала.

Последовательное подключение.

Примеры расчета

Давайте рассмотрим пример. Цепь представлена на рисунке выше. Есть источник тока и два сопротивления. Пусть R1=1,2 кОм, R2= 800 Ом, а ток в цепи 2 А. По закону Ома U = I * R. Подставляем наши значения:

  • U1 = R1 * I = 1200 Ом * 2 А = 2400 В;
  • U2 = R2 * I = 800 Ом * 2А = 1600 В.

Будет интересно➡  SMD резисторы: что это такое и для чего используются?

Общее напряжение цепи считается как сумма напряжений на резисторах: U = U1 + U2 = 2400 В + 1600 В = 4000 В. Полученную цифру можно проверить. Для этого найдем суммарное сопротивление цепи и умножим его на ток.   R = R1 + R2 = 1200 Ом + 800 Ом = 2000 Ом.

Если подставить в формулу напряжения при последовательном соединении сопротивлений, получаем: U = R * I = 2000 Ом * 2  А = 4000 В. Получаем, что общее напряжение данной цепи 4000 В.

А теперь посмотрите на схему. На первом вольтметре (возле резистора R1) показания будут 2400 В, на втором  — 1600 В.  При этом напряжение источника питания — 4000 В. Последовательное соединение – это соединение двух или более резисторов в форме цепи, в которой каждый отдельный резистор соединяется с другим отдельным резистором только в одной точке.

Материал по теме: Как проверить варистор мультиметром.

Общее сопротивление Rобщ

При таком соединении, через все резисторы проходит один и тот же электрический ток. Чем больше элементов на данном участке электрической цепи, тем «труднее» току протекать через него. Следовательно, при последовательном соединении резисторов их общее сопротивление увеличивается, и оно равно сумме всех сопротивлений.

Параллельное соединение резисторов

Параллельное соединение – это соединение, при котором резисторы соединяются между собой обоими контактами. В результате к одной точке (электрическому узлу) может быть присоединено несколько резисторов.

Параллельное подключение резисторов.

Общее сопротивление Rобщ

При таком соединении, через каждый резистор потечет отдельный ток. Сила данного тока будет обратно пропорциональна сопротивлению резистора. В результате общая проводимость такого участка электрической цепи увеличивается, а общее сопротивление в свою очередь уменьшается.

Таким образом, при параллельном подсоединении резисторов с разным сопротивлением, общее сопротивление будет всегда меньше значения самого маленького отдельного резистора.

Если посмотреть на изображение параллельного соединения, заметно, что ко всем элементам прилагается одинаковое напряжение.

То есть, при параллельном соединении резисторов, на каждом из них будет одинаковое напряжение U = U1 = U2 = U3. Получается, что ток разделяется на несколько «ручейков».

То есть, при параллельном соединении резисторов сила тока, протекающего через каждый из элементов, отличается. I = I1+I2+I3. И зависит сила тока (согласно тому же закону Ома) от сопротивления каждого участка цепи.

В случае с параллельным соединением резисторов — от их номинала.

Предлагаем также почитать интересный материал про малоизвестные факты о двигателях постоянного тока в другой нашей статье.

Схема параллельного соединения

Общее сопротивление участка цепи при таком соединении становится ниже. Его высчитывают по формуле: 1/R = 1/R1 + 1/R + 1/R3+. Такая форма хоть и понятна, но неудобна.

Формула расчета сопротивления параллельно подключенных резисторов получается тем сложнее, чем больше элементов соединены параллельно.

Но больше двух-трех редко кто объединяет, так что на практике достаточно знать только две формулы приведенные ниже.

Если подставить значения в эти формулы, то заметим, что результат будет меньше, чем сопротивление резистора с наименьшим номиналом. Это стоит запомнить: результирующее сопротивление включенных параллельно резисторов будет ниже самого маленького номинала. Давайте сначала рассчитаем параллельное соединение двух резисторов разного номинала и посмотрим что получится.

Соединили параллельно 150 Ом и 100 Ом. Считаем результирующее: 150*100 / (150+100) = 15000/250 = 60 Ом. Если соединить 150 Ом и 50 Ом, получим: 150*50 / (150+50) = 7500 / 200 = 37,5 Ом.

 Как видим, в обоих случаях результат оказывается меньше чем самый низкий номинал соединенных деталей. Этим и пользуются, если в наличии нет сопротивления небольшого номинала.

Проблема только в том, что подбирать сложновато: надо каждый раз считать используя калькулятор.

Как высчитывать сопротивление составных резисторов

Возможно, вам будет проще, если знать, что соединив два одинаковых резистора параллельно, получим результат в два раза меньше. Например, соединив параллельно два резистора по 100 Ом получим составное сопротивление 50 Ом. Проверим? Считаем: 100*100 / (100+100) = 10000 / 200 = 50 Ом. При соединении параллельно трех резисторов, считать приходится больше, так как формула сложнее.

Если подключить параллельно 150 Ом, 100 Ом и 50 Ом, результирующее будет 27,3 Ом. Попробуем с более низкими номиналами. Если параллельно включены 20 Ом, 15 Ом и 10 Ом. Получим результирующее сопротивление 4,61 Ом. Вот вам подтверждение правила. Суммарное сопротивление параллельно соединенных резисторов меньше чем самый низкий номинал.

Параллельное соединение резисторов

Параллельное соединение резисторов это соединение, в котором начала всех резисторов соединены в одну общую точку (А), а концы в другую общую точку. При этом по каждому резистору течет свой ток. При параллельном соединении при протекании тока из точки А в точку Б, он имеет несколько путей.

 Таким образом, увеличение числа параллельно соединенных резисторов ведет к увеличению путей протекания тока, то есть к уменьшению противодействия протеканию тока. А это значит, чем большее количество резисторов соединить параллельно, тем меньше станет значение общего сопротивления такого участка цепи.

Общее сопротивление параллельно соединенных резисторов определяется следующим отношением: 1/Rобщ= 1/R1+1/R2+1/R3+…+1/Rn.

Формулы расчета параллельного и последовательного подключения.

Следует отметить, что здесь действует правило «меньше – меньшего». Это означает, что общее сопротивление всегда будет меньше сопротивления любого параллельно включенного резистора. Общее сопротивление для двух параллельно соединенных резисторов рассчитывается по следующей формуле: Rобщ= R1*R2/R1+R2.

Если имеет место два параллельно соединенных резистора с одинаковыми сопротивлениями, то их общее сопротивление будет равно половине сопротивления одного из них. Параллельное соединение резисторов. При параллельном соединении резисторов нескольких приемников они включаются между двумя точками электрической цепи, образуя параллельные ветви.

Заменяя лампы резисторами с сопротивлениями R1, R2, R3, При параллельном соединении ко всем резисторам приложено одинаковое напряжение U. Поэтому согласно закону Ома: I1=U/R1; I2=U/R2; I3=U/R3.

Источник: https://ElectroInfo.net/radiodetali/rezistory/kak-otlichaetsja-parallelnoe-i-posledovatelnoe-soedinenie-rezistorov.html

Расчет сопротивления двух параллельно соединенных резисторов. Последовательное и параллельное соединение резисторов. Формула для расчета параллельного соединения сопротивлений

В каждой электрической схеме присутствует резистор, имеющий сопротивление электрическому току. Резисторы бывают двух типов: постоянные и переменные. Во время разработки любой электрической схемы и ремонта электронных изделий часто приходится применять резистор, обладающий необходимым номиналом.

Несмотря на то что для резисторов предусмотрены различные номиналы , может случиться так, что не будет возможности найти необходимый или же вообще ни один элемент не сможет обеспечить требуемый показатель.

Рассчитать производительность и работу

Угол сдвига фаз вычисляется по изображению указателя. Чтобы иметь возможность определять мощности, поглощаемые схемой, предыдущие формулы используются снова. Для определения работы используются следующие формулы.

Дальнейший интересный контент по теме
Резисторы переменного тока представляют собой омические, индуктивные и емкостные резисторы. Для параллельного подключения таких резисторов в цепи переменного тока применяются разные законы, чем для сопротивлений в цепи постоянного тока. Учитывая это, например, катушку: настоящая катушка имеет как индуктивное, так и омическое сопротивление и поэтому может рассматриваться как последовательная связь чисто индуктивного и чисто омического резистора.

Решением этой проблемы может стать применение последовательного и параллельного соединения. Ознакомившись с этой статьей, вы узнаете об особенностях выполнения расчета и подбора различных номиналов сопротивлений.

Часто при изготовлении какого-либо устройства используют резисторы, которые соединяются в соответствии с последовательной схемой. Эффект от применения такого варианта сборки сводится к увеличению общего сопротивления цепи. Для данного варианта соединения элементов создаваемое ими сопротивление рассчитывается как сумма номиналов. Если же сборка деталей выполняется по параллельной схеме, то здесь потребуется рассчитать сопротивление , используя нижеописанные формулы.

Примеры параллельного соединения проводников

Мы рассматриваем только индуктивную составляющую резистора, т.е. катушку как чисто индуктивный резистор. Аналогично, используется омическое сопротивление и емкостное сопротивление, поскольку омическое сопротивление также может иметь индуктивный компонент. В то время как в случае сопротивления проволоки витки, подобные виткам, видны напрямую, это обычно скрыто в резисторах слоя. Фактически, проводящий слой наносят на носитель, из которого материал, проводящий материал, удаляется с помощью процесса спирально-циркулирующего фрезерования, так что остается спирально циркулирующий слой.

К схеме параллельного соединения прибегают в ситуации, когда стоит задача по снижению суммарного сопротивления, а, помимо этого, увеличения мощности для группы элементов, подключенных по параллельной схеме, которое должно быть больше, чем при их отдельном подключении.

Таким образом генерируется требуемое значение сопротивления. Сразу видно, что эта катушечная структура приводит к индуктивному компоненту. Однако это обычно настолько мало, что его можно пренебречь. Общая обработка взаимосвязи любых резисторов переменного тока невозможна и не требуется с помощью математических знаний, доступных в школе.

Объяснение Подключение серии и параллельное соединение

Ниже приведен упрощенный случай параллельной схемы чисто омического, индуктивного и емкостного резисторов. В этой статье мы рассмотрим параллельное соединение и последовательное соединение резисторов. Давайте сначала уточним, что такое последовательное соединение и что такое параллельное соединение, и где разница между последовательным соединением и параллельным соединением. В последовательной цепи мы имеем два или более сопротивления последовательно. Тот же ток протекает через все резисторы.

Расчет сопротивления

В случае подключения деталей друг с другом, с применением параллельной схемы для расчета суммарного сопротивления, будет использоваться следующая формула:

R(общ)=1/(1/R1+1/R2+1/R3+1/Rn).

  • R1- R3 и Rn – резисторы, подсоединенные по параллельной схеме.

Причем, если цепь создается на основе только двух элементов, то для определения суммарного номинального сопротивления следует использовать такую формулу:

Универсальная схема расчета

На следующем графике показаны резисторы последовательно, два резистора, индивидуально нарисованные в начале, и три резистора под электрической цепью. Напротив, существует параллельное соединение резисторов. Что такое параллельная схема? Теперь, в параллельной цепи, линия распадается, и, следовательно, и ток разлагается. В случае параллельного подключения резисторов во многих случаях впервые рассматривается параллельное соединение двух резисторов. Это выглядит следующим образом, включая формулу для расчета.

R(общ)=R1*R2/R1+R2.

  • R(общ) – суммарное сопротивление;
  • R1 и R2 – резисторы, подсоединенные по параллельной схеме.

Универсальная схема расчета

Применительно к радиотехнике следует уделить внимание одному важному правилу: если подключаемые друг к другу элементы по параллельной схеме имеют одинаковый показатель , то для расчета суммарного номинала необходимо общее значение разделить на число подключенных узлов:

Для трех резисторов в параллельной схеме это будет выглядеть на следующем графике, включая формулу для расчета. Разностное соединение и параллельное соединение. В случае последовательной цепи все резисторы подключаются по одной линии за другой. В случае параллельной схемы, с другой стороны, линия расщепляется, резисторы лежат в отдельных линиях. В последовательной цепи тот же ток протекает через все резисторы, а в случае параллельной цепи ток расщепляется. В случае параллельной схемы одно и то же напряжение подается на каждый резистор, но не в последовательной цепи. Другое примечание: смесь последовательной цепи и параллельной схемы называется групповой схемой.

  • R(общ) – суммарное значение сопротивления;
  • R – номинал резистора, подсоединенного по параллельной схеме;
  • n – число подключенных узлов.

Особое внимание следует обратить на то, что конечный показатель сопротивления в случае использования параллельной схемы подключения обязательно будет меньше по сравнению с номиналом любого элемента, подключаемого в цепь.

Примеры Подключение серий и параллельное соединение

В следующих примерах мы увидим, как рассчитать смесь схемы параллельной цепи и серии. В этой области шаг за шагом должен быть рассчитан набор последовательных схем и параллельных схем. На следующем графике показана смесь последовательного соединения и параллельного соединения. Каково общее сопротивление?

Сначала мы суммируем 20 Ом и 30 Ом, так как здесь имеется параллельная схема. Таким образом, схема выглядит следующим образом. Теперь добавим эту схему, добавив резисторы для вычисления общего сопротивления. Вычислите общее сопротивление следующего контура.

Пример расчета

Для большей наглядности можно рассмотреть следующий пример: допустим, у нас есть три резистора, чьи номиналы соответственно равны 100, 150 и 30 Ом. Если воспользоваться первой формулой для определения общего номинала, то получим следующее:

Прежде всего, вы должны увидеть, что есть короткое замыкание на резисторе с 95 Ом. Поэтому ток течет практически полностью по линии ниже, а 95 Ом не учитывается при расчете полного сопротивления. В противном случае у нас есть сочетание последовательной цепи и параллельной схемы.

Комбинированные последовательные и параллельные схемы

Вам нужно знать, как рассчитать резисторы последовательно, параллельно и комбинацию резисторов параллельно и последовательно? Если вы не хотите жарить свою печатную плату, вы это делаете! Эта статья покажет вам, как это сделать за несколько простых шагов. Это просто образный способ говорить, чтобы понятий было легко понять.

Некоторые факты, которые вы должны учитывать
Любой материал, который проводит электрический ток, имеет удельное сопротивление, которое представляет собой сопротивление материала при прохождении электрического тока.
  • Понять понятие сопротивления.
  • Единицей измерения резисторов является Ом.
Параллельное соединение резисторов характеризуется тем, что входные клеммы каждого из резисторов соединены друг с другом.

R(общ)=1/(1/100+1/150+1/30)=1/(0,01+0,007+0,03)=1/0,047=21,28Ом.

Если выполнить несложные расчеты, то можно получить следующее: для цепи, включающей в себя три детали, где наименьший показатель сопротивления составляет 30 Ом, результирующее значение номинала будет равно 21,28 Ом. Этот показатель будет меньше минимального значения номинала в цепи практически на 30%.

Аналогично, в конфигурации параллельного резистора выходные клеммы также соединены друг с другом. Из-за этого все резисторы пропускают одинаковое напряжение, т.е. имеют одинаковое падение напряжения. Это связано с тем, что концы каждого из резисторов соединены с одной и той же точкой в ​​цепи, и поэтому они имеют одинаковое напряжение.

Однако общий ток, протекающий через резисторы параллельно, равен сумме интенсивностей, которые проходят каждый резистор. Дифференциация параллельного соединения из последовательного интерфейса проста. В последовательной конфигурации резистора выходной разъем одного подключается к входному разъему следующего.

Важные нюансы

Обычно для резисторов параллельное соединение применяется тогда, когда стоит задача по созданию сопротивления большей мощности. Для ее решения потребуются резисторы, которые должны иметь равные показатели сопротивления и мощности. При таком варианте определить общую мощность можно следующим образом : мощность одного элемента необходимо перемножить с суммарным числом всех резисторов, из которых состоит цепь, подсоединенных друг с другом в соответствии с параллельной схемой.

Вычисление сопротивлений параллельно: формула

Чтобы вычислить эквивалентное сопротивление нескольких подключенных параллельных резисторов, мы должны применить формулу, указанную выше этих строк. Чтобы избежать ошибок в расчетах, лучше всего разделить формулу на два шага. Сначала мы вычисляем сумму обратного для каждого сопротивления и, когда получаем результат, вычисляем его обратно, чтобы знать эквивалентное сопротивление.

Решенное сопротивление сопротивлениям параллельно

Например, мы вычислим эквивалентное сопротивление конфигурации, аналогичное той, что мы имеем на следующем рисунке. Первый шаг: вычислить сумму обратного каждого сопротивления. Шаг второй: вычислите обратное только что полученное сопротивление.

Вычисление трех резисторов параллельно
Если мы хотим решить предыдущий пример, но используя наш калькулятор из трех резисторов параллельно в сети, просто заполните значение каждого резистора в соответствующем поле. Порядок, в котором вы его пишете, не имеет значения, поэтому вам не нужно его уважать.

Скажем, если нами будут использоваться пять резисторов, чей номинал составляет 100 Ом, а мощность каждого равна 1 Вт, которые присоединены друг к другу в соответствии с параллельной схемой, то суммарный показатель сопротивления будет равен 20 Ом, а мощность составит 5 Вт.

Если взять те же резисторы, но подсоединить их в соответствии с последовательной схемой, то конечная мощность составит 5 Вт, а суммарный номинал будет равен 500 Ом.

Когда вы пишете значение трех резисторов параллельно, просто нажмите кнопку расчета, и вы автоматически получите результат без применения формулы для расчета сопротивления параллельно. С этим вы экономите время и, прежде всего, просчеты. Как мы видели в предыдущих разделах, устройства, которые выступают против прохода электрического тока более выраженным образом, чем обычно, обычно используются в электрических цепях. Эти устройства называются резисторами и могут быть связаны таким образом, что вместе они эквивалентны значению другого сопротивления, называемого эквивалентным сопротивлением.

Заключение

Параллельная схема подключения резисторов очень востребована по той причине, что часто возникает задача по созданию такого номинала, которого невозможно добиться при помощи простого параллельного соединения. При этом процедура расчета этого параметра отличается достаточной сложностью , где необходимо учитывать разные параметры.

Ассоциация резисторов в серии

Он называется полученным сопротивлением или эквивалентом, к значению сопротивления, которое получается путем связывания их набора. В основном резисторы могут быть связаны последовательно, параллельно или комбинацией обоих смешанных вызовов. Когда два или более резисторов последовательно, интенсивность тока, проходящая через каждую из них, одинакова.

Если применить закон Ома к каждому из сопротивлений предыдущего рисунка, мы получим. Если мы сделаем сумму от члена к элементу по трем уравнениям, заметим, что. Таким образом, приведенное выше уравнение, если учесть, что. Итак, если вы понимаете, вы можете видеть, что три предыдущих резистора серии эквивалентны одному резистору, значение которого представляет собой сумму трех предыдущих.

Здесь важная роль отводится не только количеству подключаемых элементов, но и рабочим параметрам резисторов — прежде всего, сопротивлению и мощности. Если один из подключаемых элементов будет иметь неподходящий показатель, то это не позволит эффективно решить задачу по созданию требуемого номинала в цепи.

Последовательное соединение это соединение двух или более резисторов в форме цепи, в которой каждый отдельный резистор соединяется с другим отдельным резистором только в одной точке.

Ассоциация резисторов в параллельном

Когда два или более резисторов параллельны, они делятся своими концами, как показано на следующем рисунке. Если мы применим закон Ома в каждом из сопротивлений фигуры. Зная, что сумма интенсивностей каждого сопротивления равна интенсивности перед входом и выходом из набора, образованного тремя сопротивлениями.

Ассоциация смешанного сопротивления

Как правило, в электрических цепях они не просто похожи на последовательные или параллельные резисторы, но и на комбинацию обоих. Чтобы лучше понять, как подойти к этим типам ассоциаций, мы проиллюстрируем пример. Представьте себе следующую схему сопротивлений.

Общее сопротивление R общ

При таком соединении, через все резисторы проходит один и тот же электрический ток. Чем больше элементов на данном участке электрической цепи, тем «труднее» току протекать через него. Следовательно, при последовательном соединении резисторов их общее сопротивление увеличивается, и оно равно сумме всех сопротивлений.

Подключение 2 равных громкоговорителей последовательно добавляет импедансы и ватты? Сопротивление добавляется, и общая мощность рассеивания на громкоговоритель уменьшается вдвое. Предположим, что выход 8 вольт и динамик 8 Ом ток, который циркулирует громкоговорителем.

Таким образом, динамик должен будет поддерживать более 8 Вт, чтобы он не был поврежден. Теперь подключите два динамика 8 Ом и 8 Вт последовательно, импеданс обоих составляет 16 Ом. Теперь мы вычисляем ток, протекающий через динамики. С этими данными мы вычисляем мощность в каждом динамике.

Напряжение при последовательном соединении

Напряжение при последовательном соединении распределяется на каждый резистор согласно закону Ома:

Т.е чем большее сопротивление резистора, тем большее напряжение на него падает.

Вывод: динамики работают более сдержанно, но усилитель будет поставлять половину мощности, когда динамик составляет 8 Ом. Разделен ли импеданс и ватт параллельно? Если мы рассмотрим одни и те же ораторы на примере предыдущего вопроса, можно сказать, что общий импеданс уменьшается наполовину и рассчитывается следующим образом.

Для работы с более чем двумя динамиками параллельно необходимо использовать другое уравнение для расчета импеданса. Если мы анализируем отдельно каждого динамика, мы понимаем, что каждый из них ведет себя так же, как в примере 1, когда каждый из них подключен к усилителю, через который они будут циркулировать 1 А, а мощность, подлежащая рассеиванию, будет составлять 8 Вт в каждом динамике.

Параллельное соединение это соединение, при котором резисторы соединяются между собой обоими контактами. В результате к одной точке (электрическому узлу) может быть присоединено несколько резисторов.

Общее сопротивление R общ

При таком соединении, через каждый резистор потечет отдельный ток. Сила данного тока будет обратно пропорциональна сопротивлению резистора. В результате общая проводимость такого участка электрической цепи увеличивается, а общее сопротивление в свою очередь уменьшается.

Таким образом, при параллельном подсоединении резисторов с разным сопротивлением, общее сопротивление будет всегда меньше значения самого маленького отдельного резистора.

Формула общей проводимости при параллельном соединении резисторов:

Формула эквивалентного общего сопротивления при параллельном соединении резисторов:

Для двух одинаковых резисторов общее сопротивление будет равно половине одного отдельного резистора:

Соответственно, для n одинаковых резисторов общее сопротивление будет равно значению одного резистора, разделенного на n.

Напряжение при параллельном соединении

Напряжение между точками A и B является как общим напряжением для всего участка цепи, так и напряжением, падающим на каждый резистор в отдельности. Поэтому при параллельном соединении на все резисторы упадет одинаковое напряжение.

Через каждый резистор течет ток, сила которого обратно пропорциональна сопротивлению резистора. Для того чтобы узнать какой ток течет через определенный резистор, можно воспользоваться законом Ома:

Смешанным соединением называют участок цепи, где часть резисторов соединяются между собой последовательно, а часть параллельно. В свою очередь, смешанное соединение бывает последовательного и параллельного типов.

Общее сопротивление R общ

  • Цепь разбивают на участки с только пареллельным или только последовательным соединением.
  • Вычисляют общее сопротивление для каждого отдельного участка.
  • Вычисляют общее сопротивление для всей цепи смешанного соединения.

Также существует более быстрый способ расчета общего сопротивления для смешанного соединения. Можно, в соответствии схеме, сразу записывать формулу следующим образом:

  • Если резисторы соединяются последоватеьно — складывать.
  • Если резисторы соединяются параллельно — использовать условное обозначение «||».
  • Подставлять формулу для параллельного соединения где стоит символ «||».

Так это будет выглядеть для схемы 1:

Формула расчета сопротивления при параллельном соединении резисторов

Электрическое сопротивление характеризует свойство проводника препятствовать прохождению через него электрического тока. У каждого материала есть свое удельное сопротивление. Это табличная величина, и условно она считается постоянной.

Условно, потому что во многом эта характеристика зависит от внешних условий, например температуры. Сопротивление же какого-либо конкретного элемента (мы будем говорить о резисторах) складывается из многих факторов, например, из геометрических параметров, а когда речь идет о цепи переменного тока, то в расчеты включают также индуктивное и емкостное сопротивление, но об этом мы расскажем позже. Пока же — немного теории.

Закон Ома

В 1826 году немецкий физик Георг Ом на основе своих опытов вывел закон, согласно которому сила тока на участке цепи прямо пропорциональна напряжению, которое к нему приложено, и обратно пропорциональна сопротивлению участка. Из школьного курса мы знаем этот закон:

I=U/R

Позже он был сформулирован и для полной цепи:

I=ε/(R+r)

Где ε — ЭДС источника, R — сопротивление цепи, а r — сопротивление источника.2/R

Последовательное и параллельное соединение

В реальной жизни мы редко имеем дело с одним проводником и одним источником. Достаточно взглянуть в любую принципиальную электрическую схему, например, такую простенькую:

(это схема микроволновки «Электроника»)

можно увидеть, что элементы в схеме соединены по-разному, но мы покажем вам базовые закономерности, которые работают в цепях.

Правила Кирхгофа

Если взять замкнутую электрическую цепь, по которой течет заряд, то можно определенно сказать: он никуда не денется. Сумма всех зарядов, которые текут в одной цепи, всегда одинакова. Это называется законом сохранения заряда, частным случаем общего закона сохранения (как говорится, если в одном месте что-то убудет, в другом непременно прибудет).

Отсюда мы и выводим тот факт, что в каждом узле цепи сумма токов равна нулю. То есть, если ток «приходит» в точку по ветке и «уходит» по двум — значит, первый равен сумме второго и третьего.

На этой картинке мы видим, что I1+I4=I2+I3

Это называется первым правилом Кирхгофа.

Если наша цепь не будет содержать узлов, значит, ток в ней будет величиной постоянной, а элементы, один за другим поставленные в цепь, будут давать падение напряжения. При этом общее напряжение в цепи останется тем же. Отсюда вытекает второе правило Кирхгофа: сумма напряжений на участках цепи будет равна ЭДС источников тока, входящий в эту цепь. Если у нас источник один, то будет верно равенство:

ε=U1+U2+U3+…+Un

Сумма падений напряжения будет, таким образом, нулевой.

В ситуациях, когда мы имеем дело с переменным током, падение будет наблюдаться на участках с конденсаторами и катушками — в цепях переменного тока у них появляется сопротивление (об этом позже).

Теперь, когда мы познакомились с теоретической частью, можем перейти к более приближенному к суровой реальности вопросу, а именно — расчету последовательного и параллельного соединения резисторов.2/R

Исходя из вышеперечисленных закономерностей, вы сможете рассчитывать самые причудливые соединения резисторов, можете попрактиковаться, взяв в библиотеке задачник.

Типы резисторов

Как уже было сказано ранее, элемент, который ставится в цепь для нагрузки, называется резистором. Ставят его для разных целей, главным образом для того, чтобы изменить тот или иной параметр на участке цепи. Например, понизить напряжение или силу тока, чтобы деталь, стоящая за резистором, не сгорела.

Предприятиями выпускается большой ассортимент таких изделий, и их можно по-разному классифицировать. Номинально резистор имеет то сопротивление, которое указано на нем, а по факту оно может зависеть от напряжения в сети (нелинейность), иметь разброс параметра (иногда до 20% доходит). По применяемой технологии резисторы можно разделить на:

  1. проволочные;
  2. композитные;
  3. металлофольговые;
  4. угольные;
  5. интегральные.

Фактическое сопротивление такого элемента может зависеть от температуры окружающей среды и даже от частоты, если мы имеем дело с переменным током. Дело в том, что часть ассортимента резисторов выполнены по проволочной технологии, то есть фактически они представляют собой мини-катушку. При малых частотах (50 Гц) это в расчет не берется, а вот на высоких (мегагерцы) паразитная индуктивность и индуктивное сопротивление может сказаться на работе схемы. Поэтому при выборе резистора для работы с высокочастотными схемами внимательно смотрите. по какой технологии он сделан. Отдайте предпочтение тонкослойным и композиционным изделиям.

Помимо этого, большое распространение получили переменные резисторы, значение сопротивления которых можно регулировать. Делается это чаще всего отверткой. Необходимость в таких изделиях продиктована разбросом параметров у обычных резисторов, а подстроечный вариант позволяет регулировать сопротивление.

Все вышесказанное актуально для цепей постоянного тока и переменного при невысоких частотах, и все это — при нормальных условиях внешней среды. Расчеты цепей при нарушении этих условий нуждаются в дополнительной корректировке: это связано с ограниченностью действия закона Ома. С чем связаны ограничения? Вот несколько примеров:

  1. при сверхнизких температурах многие проводники проявляют такое интересное явление, как сверхпроводимость;
  2. также сопротивление может разниться при нагревании;
  3. неприменим закон Ома для описания электрического тока в газах;
  4. наконец, обычный резистор можно просто пробить высоким напряжением.

Все это прекрасно работает. Не верите — можете поэкспериментировать у себя дома или провести замеры тестером. Например, изучить елочную гирлянду или показания счетчиков при включенных электроприборах (напомню, что в гирлянде лампочки соединены последовательно, а розетки в доме — параллельно). Удачи!

Как я и обещал в статье про переменные резисторы (ссылка), сегодня речь пойдет о возможных способах соединения резисторов, в частности о последовательном соединении и о параллельном.

Последовательное соединение резисторов.

Давайте начнем с рассмотрения цепей, элементы которой соединены последовательно. И хоть мы и будем рассматривать только резисторы в качестве элементов цепи в данной статье, но правила, касающиеся напряжений и токов при разных соединениях будут справедливы и для других элементов. Итак, первая цепь, которую мы будем разбирать выглядит следующим образом:

Здесь у нас классический случай последовательного соединения – два последовательно включенных резистора. Но не будем забегать вперед и рассчитывать общее сопротивление цепи, а для начала рассмотрим все напряжения и токи. Итак, первое правило заключается в том, что протекающие по всем проводникам токи при последовательном соединении равны между собой:

А для определения общего напряжения при последовательном соединении, напряжения на отдельных элементах необходимо просуммировать:

В то же время, по закону Ома для напряжений, сопротивлений и токов в данной цепи справедливы следующие соотношения:

Тогда для вычисления общего напряжения можно будет использовать следующее выражение:

Но для общего напряжение также справедлив закон Ома:

Здесь – это общее сопротивление цепи, которое исходя из двух формул для общего напряжения равно:

Таким образом, при последовательном соединении резисторов общее сопротивление цепи будет равно сумме сопротивлений всех проводников.

Например для следующей цепи:

Общее сопротивление будет равно:

Количество элементов значения не имеет, правило, по которому мы определяем общее сопротивление будем работать в любом случае 🙂 А если при последовательном соединении все сопротивления равны (), то общее сопротивление цепи составит:

в данной формуле равно количеству элементов цепи.

С последовательным соединением резисторов мы разобрались, давайте перейдем к параллельному.

Параллельное соединение резисторов.

При параллельном соединении напряжения на проводниках равны:

А для токов справедливо следующее выражение:

То есть общий ток разветвляется на две составляющие, а его значение равно сумме всех составляющих. По закону Ома:

Подставим эти выражения в формулу общего тока:

А по закону Ома ток:

Приравниваем эти выражения и получаем формулу для общего сопротивления цепи:

Данную формулу можно записать и несколько иначе:

Таким образом, при параллельном соединении проводников величина, обратная общему сопротивлению цепи, равна сумме величин, обратных сопротивлениям параллельно включенных проводников.

Аналогичная ситуация будет наблюдаться и при большем количестве проводников, соединенных параллельно:

Смешанное соединение резисторов.

Помимо параллельного и последовательного соединений резисторов существует еще смешанное соединение. Из названия уже понятно, что при таком соединении в цепи присутствуют резисторы, соединенные как параллельно, так и последовательно. Вот пример такой цепи:

Давайте рассчитаем общее сопротивление цепи. Начнем с резисторов и – они соединены параллельно. Мы можем рассчитать общее сопротивление для этих резисторов и заменить их в схеме одним единственным резистором :

Теперь у нас образовались две группы последовательно соединенных резисторов:

Заменим эти две группы двумя резисторами, сопротивление которых равно:

Как видите, схема стала уже совсем простой ) Заменим группу параллельно соединенных резисторов и одним резистором :

И в итоге у нас на схеме осталось только два резистора соединенных последовательно:

Общее сопротивление цепи получилось равным:

Таким вот образом достаточно большая схема свелась к простейшему последовательному соединению двух резисторов 😉

Тут стоит отметить, что некоторые схемы невозможно так просто преобразовать и определить общее сопротивление – для таких схем нужно использовать правила Кирхгофа, о которых мы обязательно поговорим в будущих статьях. А сегодняшняя статья на этом подошла к концу, до скорых встреч на нашем сайте!

Проверим справедливость показанных здесь формул на простом эксперименте.

Возьмём два резистора МЛТ-2 на 3 и 47 Ом и соединим их последовательно. Затем измерим общее сопротивление получившейся цепи цифровым мультиметром. Как видим оно равно сумме сопротивлений резисторов, входящих в эту цепочку.


Замер общего сопротивления при последовательном соединении

Теперь соединим наши резисторы параллельно и замерим их общее сопротивление.


Измерение сопротивления при параллельном соединении

Как видим, результирующее сопротивление (2,9 Ом) меньше самого меньшего (3 Ом), входящего в цепочку. Отсюда вытекает ещё одно известное правило, которое можно применять на практике:

При параллельном соединении резисторов общее сопротивление цепи будет меньше наименьшего сопротивления, входящего в эту цепь.

Что ещё нужно учитывать при соединении резисторов?

Во-первых, обязательно учитывается их номинальная мощность. Например, нам нужно подобрать замену резистору на 100 Ом и мощностью 1 Вт . Возьмём два резистора по 50 Ом каждый и соединим их последовательно. На какую мощность рассеяния должны быть рассчитаны эти два резистора?

Поскольку через последовательно соединённые резисторы течёт один и тот же постоянный ток (допустим 0,1 А ), а сопротивление каждого из них равно 50 Ом , тогда мощность рассеивания каждого из них должна быть не менее 0,5 Вт . В результате на каждом из них выделится по 0,5 Вт мощности. В сумме это и будет тот самый 1 Вт .

Данный пример достаточно грубоват. Поэтому, если есть сомнения, стоит брать резисторы с запасом по мощности.

Подробнее о мощности рассеивания резистора читайте .

Во-вторых, при соединении стоит использовать однотипные резисторы, например, серии МЛТ. Конечно, нет ничего плохого в том, чтобы брать разные. Это лишь рекомендация.

Параллельное соединение резисторов — одно из двух видов электрических соединений, когда оба вывода одного резистора соединены с соответствующими выводами другого резистора или резисторов. Зачастую или параллельно для того, чтобы создать более сложные электронные схемы.

Схема параллельного соединения показан на рисунке ниже. При параллельном соединении резисторов, напряжение на всех резисторах будет одинаковым, а протекающий через них ток будет пропорционален их сопротивлению:

Формула параллельного соединения резисторов

Общее сопротивление нескольких резисторов соединенных параллельно определяется по следующей формуле:

Ток, протекающий через отдельно взятый резистор, согласно , можно найти по формуле:

Параллельное соединение резисторов — расчет

Пример №1

При разработке устройства, возникла необходимость установить резистор с сопротивлением 8 Ом. Если мы просмотрим весь номинальный ряд стандартных значений резисторов, то мы увидим, что резистора с сопротивлением в 8 Ом в нем нет.

Выходом из данной ситуации будет использование двух параллельно соединенных резисторов. Эквивалентное значение сопротивления для двух резисторов соединенных параллельно рассчитывается следующим образом:

Данное уравнение показывает, что если R1 равен R2, то сопротивление R составляет половину сопротивления одного из двух резисторов. При R = 8 Ом, R1 и R2 должны, следовательно, иметь значение 2 × 8 = 16 Ом.
Теперь проведем проверку, рассчитав общее сопротивление двух резисторов:

Таким образом, мы получили необходимое сопротивление 8 Ом, соединив параллельно два резистора по 16 Ом.

Пример расчета №2

Найти общее сопротивление R из трех параллельно соединенных резисторов:

Общее сопротивление R рассчитывается по формуле:

Этот метод расчета может быть использованы для расчета любого количества отдельных сопротивлений соединенных параллельно.

Один важный момент, который необходимо запомнить при расчете параллельно соединенных резисторов – это то, что общее сопротивление всегда будет меньше, чем значение наименьшего сопротивления в этой комбинации.

Как рассчитать сложные схемы соединения резисторов

Более сложные соединения резисторов могут быть рассчитаны путем систематической группировки резисторов. На рисунке ниже необходимо посчитать общее сопротивление цепи, состоящей из трех резисторов:


Для простоты расчета, сначала сгруппируем резисторы по параллельному и последовательному типу соединения.
Резисторы R2 и R3 соединены последовательно (группа 2). Они в свою очередь соединены параллельно с резистором R1 (группа 1).

Последовательное соединение резисторов группы 2 вычисляется как сумма сопротивлений R2 и R3:

В результате мы упрощаем схему в виде двух параллельных резисторов. Теперь общее сопротивление всей схемы можно посчитать следующим образом:

Расчет более сложных соединений резисторов можно выполнить используя законы Кирхгофа.

Ток, протекающий в цепи параллельно соединенных резисторах

Общий ток I протекающий в цепи параллельных резисторов равняется сумме отдельных токов, протекающих во всех параллельных ветвях, причем ток в отдельно взятой ветви не обязательно должен быть равен току в соседних ветвях.

Несмотря на параллельное соединение, к каждому резистору приложено одно и то же напряжение. А поскольку величина сопротивлений в параллельной цепи может быть разной, то и величина протекающего тока через каждый резистор тоже будет отличаться (по определению закона Ома).

Рассмотрим это на примере двух параллельно соединенных резисторов. Ток, который течет через каждый из резисторов (I1 и I2) будет отличаться друг от друга поскольку сопротивления резисторов R1 и R2 не равны.
Однако мы знаем, что ток, который поступает в цепь в точке «А» должен выйти из цепи в точке «B» .

Первое правило Кирхгофа гласит: «Общий ток, выходящий из цепи равен току входящий в цепь».

Таким образом, протекающий общий ток в цепи можно определить как:

Затем с помощью закона Ома можно вычислить ток, который протекает через каждый резистор:

Ток, протекающий в R1 = U ÷ R1 = 12 ÷ 22 кОм = 0,545 мА

Ток, протекающий в R 2 = U ÷ R2 = 12 ÷ 47 кОм = 0,255 мА

Таким образом, общий ток будет равен:

I = 0,545 мА + 0,255 мА = 0,8 мА

Это также можно проверить, используя закон Ома:

I = U ÷ R = 12 В ÷ 15 кОм = 0,8 мА (то же самое)

где 15кОм — это общее сопротивление двух параллельно соединенных резисторов (22 кОм и 47 кОм)

И в завершении хочется отметить, что большинство современных резисторов маркируются цветными полосками и назначение ее можно узнать .

Параллельное соединение резисторов — онлайн калькулятор

Чтобы быстро вычислить общее сопротивление двух и более резисторов, соединенных параллельно, вы можете воспользоваться следующим онлайн калькулятором:

Подведем итог

Когда два или более резистора соединены так, что оба вывода одного резистора соединены с соответствующими выводами другого резистора или резисторов, то говорят, что они соединены между собой параллельно. Напряжение на каждом резисторе внутри параллельной комбинации одинаковое, но токи, протекающие через них, могут отличаться друг от друга, в зависимости от величины сопротивлений каждого резистора.

Эквивалентное или полное сопротивление параллельной комбинации всегда будет меньше минимального сопротивления резистора входящего в параллельное соединение.

На практике нередко встречается задача нахождения сопротивления проводников и резисторов при различных способах соединения. В статье рассмотрено, как рассчитывается сопротивление при и некоторые другие технические вопросы.

Сопротивление проводника

Все проводники имеют свойство препятствовать течению электрического тока, его принято называть электрическим сопротивлением R, оно измеряется в омах. Это основное свойство проводниковых материалов.

Для ведения электротехнических расчётов применяется удельное сопротивление – ρ Ом·м/мм 2 . Все металлы – хорошие проводники, наибольшее применение получили медь и алюминий, гораздо реже применяется железо. Лучший проводник – серебро, оно применяется в электротехнической и электронной промышленности. Широко распространены сплавы с высоким значением сопротивления.

При расчёте сопротивления используется известная из школьного курса физики формула:

R = ρ · l/S, S – площадь сечения; l – длина.

Если взять два проводника, то их сопротивление при параллельном соединении станет меньше из-за увеличения общего сечения.

и нагрев проводника

Для практических расчётов режимов работы проводников применяется понятие плотности тока – δ А/мм 2 , она вычисляется по формуле:

δ = I/S, I – ток, S – сечение.

Ток, проходя по проводнику, нагревает его. Чем больше δ, тем сильнее нагревается проводник. Для проводов и кабелей разработаны нормы допустимой плотности, которые приводятся в Для проводников нагревательных устройств существуют свои нормы плотности тока.

Если плотность δ выше допустимой, может произойти разрушение проводника, например, при перегреве кабеля у него разрушается изоляция.

Правилами регламентируется производить расчёт проводников на нагрев.

Способы соединения проводников

Любой проводник гораздо удобнее изображать на схемах как электрическое сопротивление R, тогда их легко читать и анализировать. Существует всего три способа соединения сопротивлений. Первый способ самый простой – последовательное соединение.

На фото видно, что полное сопротивление равно: R = R 1 + R 2 + R 3 .

Второй способ более сложный – параллельное соединение. Расчёт сопротивления при параллельном соединении выполняется поэтапно. Рассчитывается полная проводимость G = 1/R, а затем полное сопротивление R = 1/G.

Можно поступить и по-другому, прежде рассчитать общее сопротивление при R1 и R2, после этого повторить операцию и найти R.

Третий способ соединения наиболее сложный – смешанное соединение, то есть присутствуют все рассмотренные варианты. Схема приведена на фото.

Для расчёта этой схемы её следует упростить, для этого заменяют резисторы R2 и R3 одним R2,3. Получается несложная схема.

R2,3,4 = R2,3 · R4/(R2,3 + R4).

Схема становится ещё проще, в ней остаются резисторы, имеющие последовательное соединение. В более сложных ситуациях используется этот же метод преобразования.

Виды проводников

В электронной технике, при производстве проводники представляют собою тонкие полоски медной фольги. Ввиду малой длины сопротивление у них незначительно, им во многих случаях можно пренебречь. Для этих проводников сопротивление при параллельном соединении уменьшается вследствие увеличения сечения.

Большой раздел проводников представляют обмоточные провода. Они выпускаются разных диаметров – от 0,02 до 5,6 миллиметра. Для мощных трансформаторов и электродвигателей выпускаются медные шинки прямоугольного сечения. Иногда при ремонте заменяют провод большого диаметра на несколько параллельно соединённых меньшего размера.

Особый раздел проводников представляют провода и кабели, промышленность предоставляет широчайший выбор марок для самых различных нужд. Нередко приходится заменять один кабель на несколько, меньшего сечения. Причины этого бывают самые различные, например, кабель сечением 240 мм 2 очень трудно прокладывать по трассе с крутыми изгибами. Его заменяют на 2×120 мм 2 , и проблема решена.

Расчёт проводов на нагрев

Проводник нагревается протекающим током, если его температура превысит допустимую, наступает разрушение изоляции. ПУЭ предусматривает расчёт проводников на нагрев, исходными данными для него являются сила тока и условия внешней среды, в которой проложен проводник. По этим данным из таблиц в ПУЭ выбирается рекомендуемое проводника или кабеля).

На практике встречаются ситуации, когда нагрузка на действующий кабель сильно возросла. Существует два выхода ‒ заменить кабель на другой, это бывает дорого, или параллельно ему проложить ещё один, чтобы разгрузить основной кабель. В этом случае сопротивление проводника при параллельном соединении уменьшается, следовательно падает выделение тепла.

Чтобы правильно выбрать сечение второго кабеля, пользуются таблицами ПУЭ, важно при этом не ошибиться с определением его рабочего тока. В этой ситуации охлаждение кабелей будет даже лучше, чем у одного. Рекомендуется рассчитать сопротивление при параллельном соединении двух кабелей, чтобы точнее определить их тепловыделение.

Расчёт проводников на потерю напряжения

При расположении потребителя R н на большом расстоянии L от источника энергии U 1 возникает довольно большое на проводах линии. К потребителю R н поступает напряжение U 2 значительно ниже начального U 1 . Практически в качестве нагрузки выступает различное электрооборудование, подключаемое к линии параллельно.

Для решения проблемы производят расчет сопротивления при параллельном соединении всего оборудования, так находится сопротивление нагрузки R н. Далее следует определить сопротивление проводов линии.

Здесь S – сечение провода линии, мм 2 .

Каждый в этой жизни сталкивался с резисторами. Люди с гуманитарными профессиями, как и все, изучали в школе на уроках физики проводники электрического тока и закон Ома.

С резисторами также имеют дело студенты технических университетов и инженеры различных производственных предприятий. Перед всеми этими людьми, так или иначе, вставала задача расчёта электрической цепи при различных видах соединения резисторов. В данной статье речь пойдёт о расчёте физических параметров, характеризующих цепь.

Виды соединений

Резистор – пассивный элемент , присутствующий в каждой электрической цепи. Он предназначен для того, чтобы сопротивляться электрическому току. Существует два вида резисторов:

Зачем же спаивать проводники друг с другом? Например, если для какой-то электрической цепи нужно определённое сопротивление. А среди номинальных показателей нужного нет. В таком случае необходимо подобрать элементы схемы с определёнными значениями сопротивления и соединить их. В зависимости от вида соединения и сопротивлений пассивных элементов мы получим какое-то определённое сопротивление цепи. Оно называется эквивалентным. Его значение зависит от вида спайки проводников. Существует три вида соединения проводников:

Значение эквивалентного сопротивления в цепи считается достаточно легко. Однако, если резисторов в схеме очень много, то лучше воспользоваться специальным калькулятором, который считает это значение. При ведении расчёта вручную, чтобы не допускать ошибок, необходимо проверять, ту ли формулу вы взяли.

Последовательное соединение проводников

В последовательной спайке резисторы идут как бы друг за другом. Значение эквивалентного сопротивления цепи равно сумме сопротивлений всех резисторов. Особенность схем с такой спайкой заключается в том, что значение тока постоянно . Согласно закону Ома, напряжение в цепи равно произведению тока и сопротивления. Так как ток постоянен, то для вычисления напряжения на каждом резисторе, достаточно перемножить значения. После этого необходимо сложить напряжения всех резисторов, и тогда мы получим значение напряжения во всей цепи.

Расчёт очень простой. Так как с ним имеют дело в основном инженеры-разработчики, то для них не составит труда сосчитать всё вручную. Но если резисторов очень много, то проще воспользоваться специальным калькулятором.

Примером последовательного соединения проводников в быту является ёлочная гирлянда.

Параллельное соединение резисторов

При параллельном соединении проводников эквивалентное сопротивление в цепи считается по-другому. Немного сложнее, чем при последовательном.

Его значение в таких цепях равняется произведению сопротивлений всех резисторов, делённому на их сумму. А также есть и другие варианты этой формулы. Параллельное соединение резисторов всегда снижает эквивалентное сопротивление цепи. То есть, его значение всегда будет меньше, чем наибольшее значение какого-то из проводников.

В таких схемах значение напряжения постоянно . То есть значение напряжения во всей цепи равно значениям напряжений каждого из проводников. Оно задаётся источником напряжения.

Сила тока в цепи равна сумме всех токов, протекающих через все проводники. Значение силы тока, протекающего через проводник. равно отношению напряжения источника к сопротивлению этого проводника.

Примеры параллельного соединения проводников:

  1. Освещение.
  2. Розетки в квартире.
  3. Производственное оборудование.

Для расчёта схем с параллельным соединением проводников лучше пользоваться специальным калькулятором. Если в схеме много резисторов, спаянных параллельно, то гораздо быстрее вы посчитаете эквивалентное сопротивление с помощью этого калькулятора.

Смешанное соединение проводников

Этот вид соединения состоит из каскадов резисторов . Например, у нас есть каскад из 10 проводников, соединённых последовательно, и после него идёт каскад из 10 проводников, соединённых параллельно. Эквивалентное сопротивление этой схемы будет равно сумме эквивалентных сопротивлений этих каскадов. То есть, по сути, здесь последовательное соединение двух каскадов проводников.

Многие инженеры занимаются оптимизацией различных схем. Её целью является уменьшение количества элементов в схеме за счёт подбора других, с подходящими значениями сопротивлений. Сложные схемы разбиваются на несколько небольших каскадов, ведь так гораздо проще вести расчёты.

Сейчас, в двадцать первом веке, инженерам стало гораздо проще работать. Ведь несколько десятилетий назад все расчёты производились вручную. А сейчас программисты разработали специальный калькулятор для расчёта эквивалентного сопротивления цепи. В нём запрограммированы формулы, по которым ведутся расчёты.

В этом калькуляторе можно выбрать вид соединения, и потом ввести в специальные поля значения сопротивлений. Через несколько секунд вы уже увидите это значение.

Физическая формула расчета (определения) эквивалентного сопротивления в цепи

Если электрическая цепь содержит несколько резисторов, то для подсчёта её основных параметров (силы тока, напряжения, мощности) удобно все резистивные устройства заменить на одно эквивалентное сопротивление цепи. Только для него должно выполняться следующее требование: его сопротивление должно быть равным суммарному значению сопротивлений всех элементов, то есть показания амперметра и вольтметра в обычной схеме и в преобразованной не должны измениться. Такой подход к решению задач называется методом свёртывания цепи.

Метод свёртывания цепи

Внимание! Расчёт эквивалентного (общего или суммарного) сопротивления в случае последовательного или параллельного подключения выполняется по разным формулам.

Последовательное соединение элементов

В случае последовательного подключения все приборы соединяются последовательно друг с другом, а собранная цепь не имеет разветвлений.

При таком подключении сила тока, проходящая через каждый резистор, будет одинаковая, а общее падение напряжения складывается из суммарных падений напряжения на каждом из приборов.

Последовательное подключение приборов

Чтобы определить суммарное значение в этом случае, воспользуемся законом Ома, который записывается следующим образом:

I = U/R.

Из вышестоящего выражения получаем значение R:

R = U/I (1).

Поскольку при последовательном соединении:

  • I = I1 = I2 =…= IN (2),
  • U = U1 + U2 +…+ UN (3),

формула для расчёта эквивалентного сопротивления (Rобщ или Rэкв) из (1) – (3) будет иметь вид:

  • Rэкв = (U1 + U2 + …+ UN)/I,
  • Rэкв = R1 + R2 + … + RN (4).

Таким образом, если имеется N последовательно соединённых одинаковых элементов, то их можно заменить на одно устройство, у которого:

Rобщ = N·R (5).

Параллельное соединение

При таком подключении входы от всех устройств соединены в одной точке, выходы – в другой точке. Эти точки в физике и электротехнике называются узлами. На электрических схемах узлы представляют собой места разветвления проводников и обозначаются точками.

Параллельное соединение

Расчет эквивалентного сопротивления также выполняем с помощью закона Ома.

В этом случае общее значение силы тока складывается из суммы сил токов, протекающих по каждой ветви, а величина падения напряжения для каждого устройства и общее напряжение одинаковые.

Если имеются N резистивных устройств, подключенных таким образом, то:

I = I1 + I2  + … + IN (6),

U = U1 = U2 = … = UN (7).

Из выражений (1), (6) и (7) имеем:

  • Rобщ = U/(I1 + I2 + …+ IN),
  • 1/Rэкв = 1/R1 + 1/R2 +…+ 1/RN (8).

Если имеется N одинаковых резисторов, имеющих подключение данного типа, то формула (8) преобразуется следующим образом:

Rобщ = R · R / N·R = R / N (9).

Если соединены несколько катушек индуктивности, то их суммарное индуктивное сопротивление рассчитывается так же, как и для резисторов.

Расчёт при смешанном соединении устройств

В случае смешанного подключения присутствуют участки с последовательным и параллельным подключениями элементов.

При решении задачи используют метод сворачивания цепи (метод эквивалентных преобразований). Его используют для вычисления параметров в том случае, если есть один источник энергии.

Предположим, задана следующая задача. Электрическая схема (см. рис. ниже) состоит из 7 резисторов. Рассчитайте токи на всех резисторах, если имеются следующие исходные данные:

  • R1 = 1Ом,
  • R2 = 2Ом,
  • R3 = 3Ом,
  • R4 = 6Ом,
  • R5 = 9Ом,
  • R6 = 18Ом,
  • R7 = 2,8Ом,
  • U = 32В.

Электрическая схема

Из закона Ома имеем: 

I = U/R,

где R – суммарное сопротивление всех приборов.

Его будем находить, воспользовавшись методом сворачивания цепи.

Элементы R2 и R3 подключены параллельно, поэтому их можно заменить на R2,3, величину которого можно рассчитать по формуле:

R2,3= R2·R3 / (R2+R3).

R4, R5 и R6 также включены параллельно, и их можно заменить на R4,5,6, которое вычисляется следующим образом:

1/R4,5,6 = 1/R4+1/R5+1/R6.

Таким образом, схему, изображённую на картинке выше, можно заменить на эквивалентную, в которой вместо резисторов R2, R3 и R4, R5, R6 используются R2,3 и R4,5,6.

Эквивалентная схема

Согласно картинке выше, в результате преобразований получаем последовательное соединение резисторов R1, R2,3, R4,5,6 и R7.

Rобщ может быть найдено по формуле:

Rобщ = R1 + R2,3 + R4,5,6 + R7.

Подставляем числовые значения и рассчитываем R для определённых участков:

  • R2.3 = 2Ом·3Ом / (2Ом + 3Ом) = 1,2Ом,
  • 1/R4,5,6 = 1/6Ом + 1/9Ом + 1/18Ом = 1/3Ом,
  • R4,5,6 = 3Ом,
  • Rэкв = 1Ом + 1,2Ом + 3Ом + 2,8Ом= 8Ом.

Теперь, после того, как нашли Rэкв, можно вычислять значение I:

I = 32В / 8Ом = 4А.

После того, как мы получили величину общего тока, можно вычислить силу тока, протекающую на каждом участке.

Поскольку R1, R2,3, R4,5,6 и R7 соединены последовательно, то:

I1 = I2,3 = I4,5,6 = I7 = I = 4А.

На участке R2,3 напряжение находим по формуле:

  • U2,3 = I2,3·R2,3,
  • U2,3 = 4А·1,2Ом = 4,8В.

Поскольку R2 и R3 подключены параллельно, то U2,3 = U2 = U3, следовательно:

  • I2 = U2 / R2,
  • I2 = 4,8В / 2Ом = 2,4А,
  • I3 = U3 / R3,
  • I3 = 4,8В / 3Ом = 1,6А.

Проверяем правильность решения:

  • I2,3 = I2 + I3,
  • I2,3 = 2,4А + 1,6А = 4А.

На участке R4,5,б напряжение также находим, исходя из закона Ома:

  • U4,5,6 = I4,5,6·R4,5,6,
  • U4,5,6 = 4А·3Ом = 12В.

Так как R4, R5, Rб подключены параллельно друг к другу, то:

U4,5,6 = U4 = U5 = U6 = 12В.

Вычисляем I4, I5, I6:

  • I4 = U4 / R4,
  • I4 = 12В / 6Ом = 2А,
  • I5 = U5 / R5,
  • I5 = 12В / 9Ом » 1,3А,
  • I6 = U6 / R6,
  • I5 = 12В / 18Ом » 0,7А.

Проверяем правильность решения:

I4,5,6 = 2А + 1,3А + 0,7А = 4А.

Чтобы автоматизировать выполнение расчётов эквивалентных значений для различных участков цепи, можно воспользоваться сервисами сети Интернет, которые предлагают на их сайтах выполнить онлайн вычисления нужных электрических характеристик. Сервис обычно имеет встроенную специальную программу – калькулятор, которая помогает быстро выполнить расчет сопротивления цепи любой сложности.

Таким образом, использование метода эквивалентных преобразований при расчёте смешанных соединений различных устройств позволяет упростить и ускорить выполнение вычислений основных электрических параметров.

Видео

Оцените статью:

Формула расчёта общего сопротивления при параллельном соединении

Из закона Ома и первого и второго правил Кирхгофа следует:

При параллельном соединении величина обратная полному сопротивлению, равна сумме величин, обратных сопротивлений ветвей.

При параллельном соединении полное сопротивление цепи меньше самого малого из сопротивлений ветвей.

Поскольку 1/R = G, т.е. проводимость, то
при параллельном соединении электрические проводимости отдельных ветвей складываются

Параллельное соединение двух сопротивлений

При параллельном соединении двух сопротивлений формула (1) упрощается

Параллельное соединение двух сопротивлений

При параллельном соединении двух сопротивлений формула (1) упрощается

Параллельное соединение резисторов — одно из двух видов электрических соединений, когда оба вывода одного резистора соединены с соответствующими выводами другого резистора или резисторов. Зачастую резисторы соединяют последовательно или параллельно для того, чтобы создать более сложные электронные схемы.

Схема параллельного соединения резисторов показан на рисунке ниже. При параллельном соединении резисторов, напряжение на всех резисторах будет одинаковым, а протекающий через них ток будет пропорционален их сопротивлению:

Формула параллельного соединения резисторов

Общее сопротивление нескольких резисторов соединенных параллельно определяется по следующей формуле:

Ток, протекающий через отдельно взятый резистор, согласно закону Ома, можно найти по формуле:

Параллельное соединение резисторов — расчет

Пример №1

При разработке устройства, возникла необходимость установить резистор с сопротивлением 8 Ом. Если мы просмотрим весь номинальный ряд стандартных значений резисторов, то мы увидим, что резистора с сопротивлением в 8 Ом в нем нет.

Выходом из данной ситуации будет использование двух параллельно соединенных резисторов. Эквивалентное значение сопротивления для двух резисторов соединенных параллельно рассчитывается следующим образом:

Данное уравнение показывает, что если R1 равен R2, то сопротивление R составляет половину сопротивления одного из двух резисторов. При R = 8 Ом, R1 и R2 должны, следовательно, иметь значение 2 × 8 = 16 Ом.
Теперь проведем проверку, рассчитав общее сопротивление двух резисторов:

Таким образом, мы получили необходимое сопротивление 8 Ом, соединив параллельно два резистора по 16 Ом.

Пример расчета №2

Найти общее сопротивление R из трех параллельно соединенных резисторов:

Общее сопротивление R рассчитывается по формуле:

Этот метод расчета может быть использованы для расчета любого количества отдельных сопротивлений соединенных параллельно.

Один важный момент, который необходимо запомнить при расчете параллельно соединенных резисторов – это то, что общее сопротивление всегда будет меньше, чем значение наименьшего сопротивления в этой комбинации.

Как рассчитать сложные схемы соединения резисторов

Более сложные соединения резисторов могут быть рассчитаны путем систематической группировки резисторов. На рисунке ниже необходимо посчитать общее сопротивление цепи, состоящей из трех резисторов:


Для простоты расчета, сначала сгруппируем резисторы по параллельному и последовательному типу соединения.
Резисторы R2 и R3 соединены последовательно (группа 2). Они в свою очередь соединены параллельно с резистором R1 (группа 1).

Последовательное соединение резисторов группы 2 вычисляется как сумма сопротивлений R2 и R3:

В результате мы упрощаем схему в виде двух параллельных резисторов. Теперь общее сопротивление всей схемы можно посчитать следующим образом:

Расчет более сложных соединений резисторов можно выполнить используя законы Кирхгофа.

Ток, протекающий в цепи параллельно соединенных резисторах

Общий ток I протекающий в цепи параллельных резисторов равняется сумме отдельных токов, протекающих во всех параллельных ветвях, причем ток в отдельно взятой ветви не обязательно должен быть равен току в соседних ветвях.

Несмотря на параллельное соединение, к каждому резистору приложено одно и то же напряжение. А поскольку величина сопротивлений в параллельной цепи может быть разной, то и величина протекающего тока через каждый резистор тоже будет отличаться (по определению закона Ома).

Рассмотрим это на примере двух параллельно соединенных резисторов. Ток, который течет через каждый из резисторов ( I1 и I2 ) будет отличаться друг от друга поскольку сопротивления резисторов R1 и R2 не равны.
Однако мы знаем, что ток, который поступает в цепь в точке «А» должен выйти из цепи в точке «B» .

Первое правило Кирхгофа гласит: «Общий ток, выходящий из цепи равен току входящий в цепь».

Таким образом, протекающий общий ток в цепи можно определить как:

Затем с помощью закона Ома можно вычислить ток, который протекает через каждый резистор:

Ток, протекающий в R1 = U ÷ R1 = 12 ÷ 22 кОм = 0,545 мА

Ток, протекающий в R 2 = U ÷ R2 = 12 ÷ 47 кОм = 0,255 мА

Таким образом, общий ток будет равен:

I = 0,545 мА + 0,255 мА = 0,8 мА

Это также можно проверить, используя закон Ома:

I = U ÷ R = 12 В ÷ 15 кОм = 0,8 мА (то же самое)

где 15кОм — это общее сопротивление двух параллельно соединенных резисторов (22 кОм и 47 кОм)

И в завершении хочется отметить, что большинство современных резисторов маркируются цветными полосками и назначение ее можно узнать здесь.

Параллельное соединение резисторов — онлайн калькулятор

Чтобы быстро вычислить общее сопротивление двух и более резисторов, соединенных параллельно, вы можете воспользоваться следующим онлайн калькулятором:

Подведем итог

Когда два или более резистора соединены так, что оба вывода одного резистора соединены с соответствующими выводами другого резистора или резисторов, то говорят, что они соединены между собой параллельно. Напряжение на каждом резисторе внутри параллельной комбинации одинаковое, но токи, протекающие через них, могут отличаться друг от друга, в зависимости от величины сопротивлений каждого резистора.

Эквивалентное или полное сопротивление параллельной комбинации всегда будет меньше минимального сопротивления резистора входящего в параллельное соединение.

Как правильно соединять резисторы?

О том, как соединять конденсаторы и рассчитывать их общую ёмкость уже рассказывалось на страницах сайта. А как соединять резисторы и посчитать их общее сопротивление? Именно об этом и будет рассказано в этой статье.

Резисторы есть в любой электронной схеме, причём их номинальное сопротивление может отличаться не в 2 – 3 раза, а в десятки и сотни раз. Так в схеме можно найти резистор на 1 Ом, и тут же неподалёку на 1000 Ом (1 кОм)!

Поэтому при сборке схемы либо ремонте электронного прибора может потребоваться резистор с определённым номинальным сопротивлением, а под рукой такого нет. В результате быстро найти подходящий резистор с нужным номиналом не всегда удаётся. Это обстоятельство тормозит процесс сборки схемы или ремонта. Выходом из такой ситуации может быть применение составного резистора.

Для того чтобы собрать составной резистор нужно соединить несколько резисторов параллельно или последовательно и тем самым получить нужное нам номинальное сопротивление. На практике это пригождается постоянно. Знания о правильном соединении резисторов и расчёте их общего сопротивления выручают и ремонтников, восстанавливающих неисправную электронику, и радиолюбителей, занятых сборкой своего электронного устройства.

Последовательное соединение резисторов.

В жизни последовательное соединение резисторов имеет вид:


Последовательно соединённые резисторы серии МЛТ

Принципиальная схема последовательного соединения выглядит так:

На схеме видно, что мы заменяем один резистор на несколько, общее сопротивление которых равно тому, который нам необходим.

Подсчитать общее сопротивление при последовательном соединении очень просто. Нужно сложить все номинальные сопротивления резисторов входящих в эту цепь. Взгляните на формулу.

Общее номинальное сопротивление составного резистора обозначено как Rобщ.

Номинальные сопротивления резисторов включённых в цепь обозначаются как R1, R2, R3,…RN.

Применяя последовательное соединение, стоит помнить одно простое правило:

Из всех резисторов, соединённых последовательно главную роль играет тот, у которого самое большое сопротивление. Именно он в значительной степени влияет на общее сопротивление.

Так, например, если мы соединяем три резистора, номинал которых равен 1, 10 и 100 Ом, то в результате мы получим составной на 111 Ом. Если убрать резистор на 100 Ом, то общее сопротивление цепочки резко уменьшиться до 11 Ом! А если убрать, к примеру, резистор на 10 Ом, то сопротивление будет уже 101 Ом. Как видим, резисторы с малыми сопротивлениями в последовательной цепи практически не влияют на общее сопротивление.

Параллельное соединение резисторов.

Можно соединять резисторы и параллельно:


Два резистора МЛТ-2, соединённых параллельно

Принципиальная схема параллельного соединения выглядит следующим образом:

Для того чтобы подсчитать общее сопротивление нескольких параллельно соединённых резисторов понадобиться знание формулы. Выглядит она вот так:

Эту формулу можно существенно упростить, если применять только два резистора. В таком случае формула примет вид:

Есть несколько простых правил, позволяющих без предварительного расчёта узнать, каково должно быть сопротивление двух резисторов, чтобы при их параллельном соединении получить то, которое требуется.

Если параллельно соединены два резистора с одинаковым сопротивлением, то общее сопротивление этих резисторов будет ровно в два раза меньше, чем сопротивление каждого из резисторов, входящих в эту цепочку.

Это правило исходит из простой формулы для расчёта общего сопротивления параллельной цепи, состоящей из резисторов одного номинала. Она очень проста. Нужно разделить номинальное сопротивление одного из резисторов на общее их количество:

Здесь R1 – номинальное сопротивление резистора. N – количество резисторов с одинаковым номинальным сопротивлением.

Ознакомившись с приведёнными формулами, вы скажите, что все они справедливы для расчёта ёмкости параллельно и последовательно соединённых конденсаторов. Да, только в отношении конденсаторов всё действует с точностью до «наоборот”. Узнать подробнее о соединении конденсаторов можно здесь.

Проверим справедливость показанных здесь формул на простом эксперименте.

Возьмём два резистора МЛТ-2 на 3 и 47 Ом и соединим их последовательно. Затем измерим общее сопротивление получившейся цепи цифровым мультиметром. Как видим оно равно сумме сопротивлений резисторов, входящих в эту цепочку.


Замер общего сопротивления при последовательном соединении

Теперь соединим наши резисторы параллельно и замерим их общее сопротивление.


Измерение сопротивления при параллельном соединении

Как видим, результирующее сопротивление (2,9 Ом) меньше самого меньшего (3 Ом), входящего в цепочку. Отсюда вытекает ещё одно известное правило, которое можно применять на практике:

При параллельном соединении резисторов общее сопротивление цепи будет меньше наименьшего сопротивления, входящего в эту цепь.

Что ещё нужно учитывать при соединении резисторов?

Во-первых, обязательно учитывается их номинальная мощность. Например, нам нужно подобрать замену резистору на 100 Ом и мощностью 1 Вт. Возьмём два резистора по 50 Ом каждый и соединим их последовательно. На какую мощность рассеяния должны быть рассчитаны эти два резистора?

Поскольку через последовательно соединённые резисторы течёт один и тот же постоянный ток (допустим 0,1 А), а сопротивление каждого из них равно 50 Ом, тогда мощность рассеивания каждого из них должна быть не менее 0,5 Вт. В результате на каждом из них выделится по 0,5 Вт мощности. В сумме это и будет тот самый 1 Вт.

Данный пример достаточно грубоват. Поэтому, если есть сомнения, стоит брать резисторы с запасом по мощности.

Подробнее о мощности рассеивания резистора читайте тут.

Во-вторых, при соединении стоит использовать однотипные резисторы, например, серии МЛТ. Конечно, нет ничего плохого в том, чтобы брать разные. Это лишь рекомендация.

последовательное, параллельное, смешанное соединение. Расчет сопротивления

электрика, сигнализация, видеонаблюдение, контроль доступа (СКУД), инженерно технические системы (ИТС)

Резисторы между собой могут быть соединены двумя основными способами: последовательно и параллельно. Смешанное соединение резисторов является их комбинацией.

Сочетания любых соединений резисторов можно привести к одному резистору, расчетом сопротивления которого (R) мы сейчас займемся.

ПАРАЛЛЕЛЬНОЕ СОЕДИНЕНИЕ РЕЗИСТОРОВ

Давайте рассчитаем общее сопротивление такой цепи (рисунок 1). Для этого нам понадобится закон Ома — I=U/R и закон Кирхгофа — I=I1+I2+..In

С учетом этого имеем:

  • I=U/R
  • I1=U/R1
  • I2=U/R2
  • In=U/Rn
  • U/R=U/R1+U/R2+…U/Rn
  • 1/R=1/R1+1/R2+…1/Rn

Последняя формула является основной для расчета сопротивления цепи параллельно соединенных резисторов. Для двух резисторов ее можно записать более удобно: R=(R1*R2)/(R1+R2).

Отсюда следует, что в случае параллельного соединения двух одинаковых по номиналу резисторов (R1=R2) их общее сопротивление будет вдвое меньше любого из них. Это полезно помнить.

ПОСЛЕДОВАТЕЛЬНОЕ СОЕДИНЕНИЕ РЕЗИСТОРОВ

Используя уже упомянутые законы для цепи последовательно соединенных резисторов (рисунок 2) можем записать:

  • U=I*R
  • I=I1=I2=…In
  • U=U1+U2+…Un
  • I*R=I*R1+I*R2+…I*Rn
  • R=R1+R2+…Rn

То есть общее сопротивление резисторов при последовательном соединении равно сумме их сопротивлений.

СМЕШАННОЕ СОЕДИНЕНИЕ РЕЗИСТОРОВ

Такое соединение всегда можно представить как комбинацию последовательного и параллельного соединений (рис.3).

Расчет общего сопротивления цепи при этом производится поэтапно. В приведенном примере рассчитываем:

  1. последовательное сопротивление резисторов Rпосл=R1+R2
  2. параллельное соединение R=(Rпосл*R3)/(Rпосл+R3)

Безусловно, могут встретиться более сложные варианты, но методика расчета их сопротивления та же.

Несколько слов про то, когда возникает необходимость соединять резисторы тем или иным способом:

  1. Отсутствие «под рукой» резистора нужного номинала. При этом следует помнить, что погрешности резисторов будут суммироваться.

    Например, для рисунка 3.a, если фактическая погрешность R1 составляет +10%, а R2 имеет +15%, то для Rпосл она будет +25%.

    Здесь следует обращать внимание на знак, то есть для -10% и +15% в результате получим +5%.

  2. Необходимость получить большую мощность.

    Здесь надо учесть, что при одинаковых номиналах сопротивлений и мощностей соединяемых резисторов, как при последовательном, так и при параллельном их соединении итоговая мощность будет равна сумме мощностей.

    В противном случае следует ее рассчитать, используя закон Ома и формулу для определения рассеиваемой мощности P=I*U.

Про мощность и номиналы резисторов можно почитать здесь.

© 2012-2021 г. Все права защищены.

Представленные на сайте материалы имеют информационный характер и не могут быть использованы в качестве руководящих и нормативных документов


Как найти сопротивление цепи при параллельном соединении

Как правильно соединять резисторы?

О том, как соединять конденсаторы и рассчитывать их общую ёмкость уже рассказывалось на страницах сайта. А как соединять резисторы и посчитать их общее сопротивление? Именно об этом и будет рассказано в этой статье.

Резисторы есть в любой электронной схеме, причём их номинальное сопротивление может отличаться не в 2 – 3 раза, а в десятки и сотни раз. Так в схеме можно найти резистор на 1 Ом, и тут же неподалёку на 1000 Ом (1 кОм)!

Поэтому при сборке схемы либо ремонте электронного прибора может потребоваться резистор с определённым номинальным сопротивлением, а под рукой такого нет. В результате быстро найти подходящий резистор с нужным номиналом не всегда удаётся. Это обстоятельство тормозит процесс сборки схемы или ремонта. Выходом из такой ситуации может быть применение составного резистора.

Для того чтобы собрать составной резистор нужно соединить несколько резисторов параллельно или последовательно и тем самым получить нужное нам номинальное сопротивление. На практике это пригождается постоянно. Знания о правильном соединении резисторов и расчёте их общего сопротивления выручают и ремонтников, восстанавливающих неисправную электронику, и радиолюбителей, занятых сборкой своего электронного устройства.

Последовательное соединение резисторов.

В жизни последовательное соединение резисторов имеет вид:


Последовательно соединённые резисторы серии МЛТ

Принципиальная схема последовательного соединения выглядит так:

На схеме видно, что мы заменяем один резистор на несколько, общее сопротивление которых равно тому, который нам необходим.

Подсчитать общее сопротивление при последовательном соединении очень просто. Нужно сложить все номинальные сопротивления резисторов входящих в эту цепь. Взгляните на формулу.

Общее номинальное сопротивление составного резистора обозначено как Rобщ.

Номинальные сопротивления резисторов включённых в цепь обозначаются как R1, R2, R3,…RN.

Применяя последовательное соединение, стоит помнить одно простое правило:

Из всех резисторов, соединённых последовательно главную роль играет тот, у которого самое большое сопротивление. Именно он в значительной степени влияет на общее сопротивление.

Так, например, если мы соединяем три резистора, номинал которых равен 1, 10 и 100 Ом, то в результате мы получим составной на 111 Ом. Если убрать резистор на 100 Ом, то общее сопротивление цепочки резко уменьшиться до 11 Ом! А если убрать, к примеру, резистор на 10 Ом, то сопротивление будет уже 101 Ом. Как видим, резисторы с малыми сопротивлениями в последовательной цепи практически не влияют на общее сопротивление.

Параллельное соединение резисторов.

Можно соединять резисторы и параллельно:


Два резистора МЛТ-2, соединённых параллельно

Принципиальная схема параллельного соединения выглядит следующим образом:

Для того чтобы подсчитать общее сопротивление нескольких параллельно соединённых резисторов понадобиться знание формулы. Выглядит она вот так:

Эту формулу можно существенно упростить, если применять только два резистора. В таком случае формула примет вид:

Есть несколько простых правил, позволяющих без предварительного расчёта узнать, каково должно быть сопротивление двух резисторов, чтобы при их параллельном соединении получить то, которое требуется.

Если параллельно соединены два резистора с одинаковым сопротивлением, то общее сопротивление этих резисторов будет ровно в два раза меньше, чем сопротивление каждого из резисторов, входящих в эту цепочку.

Это правило исходит из простой формулы для расчёта общего сопротивления параллельной цепи, состоящей из резисторов одного номинала. Она очень проста. Нужно разделить номинальное сопротивление одного из резисторов на общее их количество:

Здесь R1 – номинальное сопротивление резистора. N – количество резисторов с одинаковым номинальным сопротивлением.

Ознакомившись с приведёнными формулами, вы скажите, что все они справедливы для расчёта ёмкости параллельно и последовательно соединённых конденсаторов. Да, только в отношении конденсаторов всё действует с точностью до «наоборот”. Узнать подробнее о соединении конденсаторов можно здесь.

Проверим справедливость показанных здесь формул на простом эксперименте.

Возьмём два резистора МЛТ-2 на 3 и 47 Ом и соединим их последовательно. Затем измерим общее сопротивление получившейся цепи цифровым мультиметром. Как видим оно равно сумме сопротивлений резисторов, входящих в эту цепочку.


Замер общего сопротивления при последовательном соединении

Теперь соединим наши резисторы параллельно и замерим их общее сопротивление.


Измерение сопротивления при параллельном соединении

Как видим, результирующее сопротивление (2,9 Ом) меньше самого меньшего (3 Ом), входящего в цепочку. Отсюда вытекает ещё одно известное правило, которое можно применять на практике:

При параллельном соединении резисторов общее сопротивление цепи будет меньше наименьшего сопротивления, входящего в эту цепь.

Что ещё нужно учитывать при соединении резисторов?

Во-первых, обязательно учитывается их номинальная мощность. Например, нам нужно подобрать замену резистору на 100 Ом и мощностью 1 Вт. Возьмём два резистора по 50 Ом каждый и соединим их последовательно. На какую мощность рассеяния должны быть рассчитаны эти два резистора?

Поскольку через последовательно соединённые резисторы течёт один и тот же постоянный ток (допустим 0,1 А), а сопротивление каждого из них равно 50 Ом, тогда мощность рассеивания каждого из них должна быть не менее 0,5 Вт. В результате на каждом из них выделится по 0,5 Вт мощности. В сумме это и будет тот самый 1 Вт.

Данный пример достаточно грубоват. Поэтому, если есть сомнения, стоит брать резисторы с запасом по мощности.

Подробнее о мощности рассеивания резистора читайте тут.

Во-вторых, при соединении стоит использовать однотипные резисторы, например, серии МЛТ. Конечно, нет ничего плохого в том, чтобы брать разные. Это лишь рекомендация.

Последовательное соединение сопротивлений

Возьмем три постоянных сопротивления R1, R2 и R3 и включим их в цепь так, чтобы конец первого сопротивления R1 был соединен с началом второго сопротивления R 2, конец второго — с началом третьего R 3, а к началу первого сопротивления и к концу третьего подведем проводники от источника т

Калькулятор параллельных резисторов

R1 + R2 = эквивалентный резистор R схема сопротивления, эквивалентная общая сумма резисторов, упрощенная комбинация = параллельная

параллельная калькуляция резисторов R1 + R2 = эквивалентный резистор R эквивалентная схема сопротивления, полная поисковая схема резистора, упрощенная совмещенная = параллельная — sengpielaudio Sengpiel Berlin


R всего Формула:
R всего = R1 × R2 / (R1 + R2)

Введите , два значения резистора , будет рассчитано третье значение параллельной цепи.
Вы даже можете ввести общее сопротивление R всего и одно известное сопротивление R 1 или R 2 .

Формула (уравнение) для расчета двух сопротивлений R 1 и R 2 , соединенных параллельно:

Расчет необходимого параллельного резистора R 2 , при R 1 и суммарное сопротивление R дается всего :

Решение формулы R итого = ( R 1 × R 2 ) / ( R 1 + R 2 ) для R 1 :
Первый шаг — очистить все дроби, умножив на
наименьшего числа. общий знаменатель, то есть R t × R 1 × R 2 … Итак, получаем:
1/ R всего = 1/ R 1 + 1/ R 2
R всего × R 1 × R 2 [1/ R всего = 1/ R 1 + 1/ R 2 ]
R 1 × R 2 = R всего × R 2 + R всего × R 1 затем соберите члены с R 1 и решите
R 1 × R 2 R всего × R 1 = R всего × R 2
R 1 ( R 2 R всего ) = R 2 × R всего 9000 9
Последний шаг:
R 1 = R 2 × R всего / ( R 2 R всего )
или:
R 2 = R 1 × R всего / ( R 1 R всего )

Примечание: Этот калькулятор также может решать другие математические задачи.Расчет резисторов параллельно
точно так же, как и вычисления, необходимые для параллельных катушек индуктивности или последовательно включенных конденсаторов.

Два резистора, включенных параллельно, и результирующее общее сопротивление: Два одинаковых значения,
также покажите уравнение, что результаты всегда равны половине. Это упрощает работу, когда
проектирование схем или прототипирование. С кепками всегда вдвое больше, потом с кепками всего
просто сложите параллельно.

• Поисковые сопротивления R 1 и R 2 , когда известно целевое сопротивление (эквивалентное сопротивление) •

Расчет: пары резисторов — калькулятор с обратной конструкцией
Поиск R 1 и R 2 с известным целевым сопротивлением

● Рассчитать несколько резисторов параллельно ●

Этот калькулятор определяет сопротивление от до 10 резисторов, включенных параллельно .
Введите значения сопротивления в поля ниже и после ввода всех значений
. нажмите кнопку «рассчитать», и результат появится в поле под этой кнопкой.
В качестве теста, если мы введем сопротивления 4, 6 и 12 Ом, ответ должен быть 2 Ом.
Примечание. При снятии флажков вручную сохраненные значения не сбрасываются. Воспользуйтесь «сбросом».

Закон Ома — калькулятор и формулы

Два резистора, включенных параллельно, и результирующее общее сопротивление
Сопротивление в диапазоне от 1 Ом до 100 Ом

R2 R1
1 1.5 2,2 3,3 4,7 6,8 10 15 22 33 47 68
1 0,5 0,6 0,69 0.77 0,83 0,87 0,91 0,93 0,95 0,97 0,98 0,99
1,5 0,6 0,75 0,89 1.03 1,14 1,22 1,30 1,36 1,40 1,43 1.45 1,46
2,2 0,69 0,89 1,1 1,32 1,50 1,66 1,82 1,92 2,0 2,06 2,10 2,13
3,3 0,77 1.03 1,32 1.65 1,94 2,22 2,48 2,70 2,87 3,00 3,08 3,14
4,7 0,83 1,14 1,50 1,94 2,35 2,78 3,20 3,58 3,87 4,12 4.27 4,39
6,8 0,87 1,22 1,66 2,22 2,78 3,40 4,05 4,68 5,19 5,64 5,94 6,18
10 0,91 1,30 1,82 2.48 3,20 4,05 5,0 6,0 6,9 7,7 8,3 8,7
15 0,93 1,36 1,92 2,70 3,58 4,68 6,0 7,50 8,9 10,3 11,4 12.2
22 0,95 1,40 2,00 2,87 3,87 5,19 6,9 8,9 11,0 13,2 15,0 16,6
33 0,97 1,43 2,06 3,0 4.12 5,64 7,7 10,3 13,2 16,5 19,4 22,2
47 0,98 1,45 2,1 3,08 4,27 5,94 8,3 11,4 15,0 19,4 23,5 27.8
68 0,99 1,46 2,13 3,14 4,39 6,18 8,7 12,2 16,6 22,2 27,8 34,0

Примечание: Этот калькулятор также может решать другие математические задачи. Расчет резисторов параллельно
точно так же, как и вычисления, необходимые для параллельных катушек индуктивности или последовательно включенных конденсаторов.

Мощность, рассеиваемая в резисторе: P = В × I , P = В 2 / R , P = I 2 × R .

Примечание: Для последовательно подключенных резисторов ток одинаков для каждого резистора,
а для резисторов, включенных параллельно, напряжение одинаково для каждого резистора.

Резисторы серии

и параллельные

  • Изучив этот раздел, вы сможете:
  • Рассчитайте значения общего сопротивления в цепях с последовательным сопротивлением.
  • Используйте соответствующие формулы для расчета сопротивления в цепях с параллельным сопротивлением.
  • • Вычисление суммы обратных величин.
  • • Произведение над суммой.
  • Рассчитайте значения общего сопротивления в последовательных / параллельных сетях.

Расчеты в последовательно- и параллельных резисторных цепях

Компоненты, включая резисторы в цепи, могут быть соединены вместе двумя способами:

ПОСЛЕДОВАТЕЛЬНО, так что один и тот же ток течет через все компоненты, но различная разность потенциалов (напряжение) может существовать на каждом из них.

ПАРАЛЛЕЛЬНО, так что одна и та же разность потенциалов (напряжение) существует на всех компонентах, но каждый компонент может проводить разный ток.

Рис. 4.2.1 Резисторы серии

Рис. 4.2.2 Параллельные резисторы

В любом случае (для резисторов) общее сопротивление той части цепи, которая содержит резисторы, может быть рассчитано с использованием методов, описанных ниже.

Возможность рассчитать суммарное (общее) значение резисторов таким способом позволяет легко вычислить неизвестные значения сопротивления, тока и напряжения для довольно сложных схем с использованием относительно простых методов.Это очень полезно при поиске неисправностей.

ПЕРЕД ДАЛЬНЕЙШЕЙ ДАЛЬНОСТЬЮ ПОПРОБУЙТЕ ИСПОЛЬЗОВАНИЕ ФОРМУЛ ДЛЯ РАСЧЕТА ОБЩИХ ЗНАЧЕНИЙ СЕРИЙНЫХ И ПАРАЛЛЕЛЬНЫХ РЕЗИСТОРОВ.

Для резисторов в серии:

Общее сопротивление двух или более резисторов, подключенных последовательно , определяется простым сложением индивидуальных значений резисторов, чтобы найти общую сумму (R TOT ):

Для резисторов, включенных параллельно:

Для расчета общего сопротивления цепи, в которой используются параллельные резисторы, можно использовать следующую формулу.

Обратите внимание, однако, что эта формула НЕ дает вам общего сопротивления R TOT . Это дает вам ВЗАИМОДЕЙСТВИЕ РИНГ или:

Это совсем другое значение — и НЕ является полным сопротивлением. Он делится на 1, деленный на TOT . Чтобы получить правильное значение для R TOT (которое будет обратным 1/ TOT , то есть TOT /1, просто нажмите соответствующую клавишу на вашем калькуляторе (отмеченную 1 / x или x-1) .

Другой способ расчета параллельных цепей.

Общее сопротивление двух резисторов, включенных параллельно , которое не включает обратные, определяется как:

Эту формулу часто называют «произведение над суммой».

Он рассчитывает только ДВА резистора параллельно? Ну да, но это не большая проблема. Если имеется более двух параллельных резисторов, просто выберите два из них и определите общее сопротивление для этих двух — затем используйте это общее сопротивление, как если бы это был один резистор, и составьте еще одну пару с третьим резистором.Определите новую сумму и так далее, пока вы не включите все параллельные резисторы в этой конкретной сети.

О, еще кое-что, что нужно помнить о произведении над суммой, видите скобки вокруг суммы (нижняя часть) формулы? Это означает, что вы должны решить это ДО того, как использовать его для разделения продукта (верхняя часть) на. Если вы этого не сделаете, ваш ответ будет неправильным.

Звучит сложно? Не совсем, это просто вопрос повторения, и на практике вы не часто встречаетесь с множеством параллельных сетей с гораздо более чем двумя резисторами.Тем не менее, какую формулу вы выберете, зависит от вас, взаимная или сумма продукта.

подсказок

Использование обратного метода

Если вы используете МЕТОД ВЗАИМОДЕЙСТВИЯ для параллельных цепей, НЕ ЗАБУДЬТЕ, когда вы добавили обратные величины отдельных резисторов — вы должны снова найти обратную величину. 1 / R1 + 1 / R2 + 1 / R3 = 1 / R TOT , и чтобы найти R TOT , вы должны найти обратную величину 1 / R TOT .

Упрощающие схемы

Для комбинированных последовательных и параллельных цепей сначала разработайте секцию цепи (последовательную или параллельную).Затем перерисуйте схему, заменив участок, сопротивление которого вы нашли, одним резистором. Теперь у вас есть упрощенная схема, по которой можно найти R TOT .

Вы можете использовать формулу «произведение на сумму»:

Для цепей с более чем двумя параллельными резисторами просто выработайте два параллельных резистора одновременно, используя формулу произведения на сумму, а затем перерисуйте схему, заменив два резистора одним резистором, значение которого является объединенным сопротивлением двух .

Теперь вы можете использовать ваше первое комбинированное значение в качестве единственного резистора со следующим параллельным резистором и так далее. Таким образом, можно выработать большое количество параллельных резисторов с использованием произведения на сумму.

Когда все параллельные резисторы одинакового номинала.

Если несколько одинаковых параллельных резисторов подключены, общее сопротивление будет равно величине резистора, умноженной на обратную величину количества резисторов.

, то есть два параллельно включенных резистора 12 кОм имеют общее сопротивление

12K x 1/2 = 6K

Три параллельно включенных резистора 12 кОм имеют суммарное сопротивление

12K x 1/3 = 4K и т. Д.

Проверяю ваш ответ

Суммарное значение любого количества параллельных резисторов всегда будет МЕНЬШЕ, чем значение наименьшего отдельного резистора в сети. Используйте этот факт, чтобы проверить свои ответы.

Серия

и параллельная комбинация

Попробуйте несколько вычислений, основанных на последовательной и параллельной цепях резисторов. Для этого вам просто нужно использовать информацию на этой странице и на странице «Советы по расчету резисторов». Вас просят вычислить общее сопротивление для каждой цепи.Вы можете выбрать, какую формулу использовать

Вы также можете получить помощь по математике, загрузив нашу бесплатную брошюру «Советы по математике».

Прежде чем начать, подумайте об этих нескольких советах. Они упростят задачу, если вы будете внимательно им следовать.

1. Разработайте ответы с помощью карандаша и бумаги; перерисуйте схему, над которой работаете.

2. Конечно, ответ — это не просто число, это будет определенное количество Ом, не забудьте указать правильную единицу (например.грамм. Ω, KΩ или MΩ) или ваш ответ не имеет смысла.

3. Когда вы вводите значения в калькулятор, преобразуйте все значения KΩ или MΩ в Ом с помощью клавиши EXP. Если вы здесь ошибетесь, то получите действительно глупые ответы, в тысячи раз слишком большие или слишком маленькие.

Итак, вы прочитали эти инструкции и готовы приступить к работе. Вот способ решить типичную проблему на бумаге, чтобы (с практикой) вы не запутались.

Пример серии

и параллельной цепи.

Хорошо, есть что вспомнить, так почему бы не попробовать несколько практических вопросов в модуле резисторов 4.5 по определению общего сопротивления некоторых цепей резисторов?

резисторов, включенных параллельно | Формула эквивалентного сопротивления

Введение

Два резистора считаются подключенными параллельно, если оба вывода резистора подключены к каждому соответствующему выводу другого резистора. В сети параллельных резисторов ток может проходить по нескольким путям, в отличие от сети с последовательными резисторами, поскольку существует несколько путей для прохождения тока.Следовательно, параллельные резистивные цепи являются делителями тока.

Параллельные резисторы

Если два или более резистора подключены параллельно, то разность потенциалов на каждом резисторе одинакова. При параллельном включении резисторы подключаются к одним и тем же узлам. Это можно определить по наличию более чем одного пути прохождения тока. Например, приведенная ниже схема представляет собой параллельное соединение резисторов. Разность потенциалов на резисторе R1 такая же, как и на резисторе R2, который равен потенциалу питания V AB .

Если V AB — это подаваемый потенциал, то

V R1 = V R2 = V AB

В следующей схеме резисторы R1, R2 и R3 подключены параллельно комбинация.

Здесь потенциал питания составляет V AB между точками A и B. Поскольку резисторы R1, R2 и R3 соединены параллельно, разность потенциалов на каждом резисторе такая же, как и у источника питания.Следовательно, V AB = V R1 = V R2 = V R3 .

Где

В R1 — потенциал на резисторе R1.

В R2 — это потенциал на резисторе R2.

В R3 — потенциал на резисторе R3.

Но ток, протекающий через эти три резистора, разный. Если I — это ток, покидающий узел A, то он имеет три пути, чтобы достичь узла B. Ток, протекающий через каждый резистор, зависит от его сопротивления.Следовательно, в случае параллельных резистивных цепей ток во всех резисторах неодинаков. Если I1 — это ток, протекающий через резистор R1, I2 — это ток, протекающий через резистор R2, а I3 — это ток, протекающий через резистор R3, тогда токи I, I1, I2 и I3 могут быть связаны с помощью закона тока Кирхгофа. . Согласно текущему закону Кирхгофа, «сумма токов, входящих в узел, равна сумме токов, выходящих из узла».

Отсюда

I = I1 + I2 + I3.

Формула эквивалентного сопротивления

Любое количество резисторов, включенных в параллельную комбинацию, может быть заменено одним резистором с сопротивлением, равным эквивалентному сопротивлению резисторов параллельной комбинации.

Было установлено, что напряжение на каждом резисторе в параллельной комбинации одинаково, а общий ток равен сумме отдельных токов. Рассмотрим следующую схему.

Здесь I = I1 + I2 + I3

I1 = V / R1

I2 = V / R2

I3 = V / R3

Если R T — полное сопротивление цепи, то

I = V / R T

Следовательно V / R T = V / R 1 + V / R 2 + V / R 3

1 / R T = 1 / R 1 + 1 / R 2 + 1 / R 3

Если R eq является эквивалентным сопротивлением цепи, то оно рассчитывается путем сложения обратных значений отдельных сопротивлений (1 / R).Обратная величина этой алгебраической суммы даст эквивалентное сопротивление. Уравнение эквивалентного сопротивления R eq показано ниже для параллельной резистивной цепи из n резисторов.

(1 / R экв ) = (1 / R1) + (1 / R2) + (1 / R3) + ……… + (1 / Rn)

Из приведенного выше уравнения можно сделать вывод, что эквивалентное сопротивление резисторов, соединенных параллельно, всегда меньше, чем сопротивление самого маленького резистора.

Если два резистора подключены параллельно, то эквивалентное сопротивление будет

(1 / R eq ) = (1 / R1) + (1 / R2)

R EQ = R 1 * R 1 / (R 1 + R 2 )

Если два резистора равного сопротивления R соединены параллельно, то эквивалентное сопротивление комбинации будет R / 2.

Аналогичным образом, если три резистора равного сопротивления R соединены в параллельном соединении, тогда эквивалентное сопротивление комбинации будет R / 3.

Параллельное соединение резисторов дает значение проводимости. Проводимость — это величина, обратная сопротивлению. Обычно он обозначается символом G. Единицы проводимости — Сименс, представленные символом S. Ранее единицами проводимости были Mho (℧), что означает обратное написание Ом, а символ — перевернутое представление Ω.

Несмотря на то, что параллельные резисторы подключены между двумя узлами, представление этого подключения может иметь любую из следующих форм.

Все вышеупомянутые комбинации представляют собой параллельные резистивные схемы, и все правила параллельных резисторов применимы и к вышеупомянутым комбинациям.

Расчет тока

Ток в каждой ветви параллельной резистивной цепи отличается от другой. Поскольку напряжение на каждом резисторе одинаково, ток, протекающий через каждый резистор, зависит от сопротивления этого резистора.Следовательно, если значение сопротивления в одной ветви отличается от другой, тогда ток в этих ветвях будет другим. Величину этого тока можно определить с помощью закона Ома.

Рассмотрим параллельную сеть из двух резисторов с напряжением питания V между двумя точками A и B.

Пусть I будет полным током в следующей цепи.

Пусть ток, протекающий через резистор R 1 , равен I R1 , а ток, протекающий через резистор R 2 , равен I R2 .

Тогда, согласно закону Кирхгофа, «полный ток, входящий в цепь, равен току, выходящему из цепи».

Если I T — это общий ток, тогда

I T = I R1 + I R2

Поскольку падение напряжения на каждом резисторе одинаково

I R1 = V / R 1

And I R2 = V / R 2

Если рассматривается параллельная резистивная цепь, состоящая из n резисторов, то общий ток в цепи составляет

I Total = I R1 + Я R2 +….+ I Rn

Если последовательные резистивные цепи называются цепями делителя напряжения, то аналогично параллельные резистивные цепи называются цепями делителя тока.

Если рассматривается параллельная резистивная цепь из n резисторов с разным сопротивлением, то можно иметь n разных путей для протекания тока и n разных значений тока через эти пути. Резисторы в параллельной комбинации можно менять местами, не влияя на общий ток и эквивалентное сопротивление.

Пример параллельного подключения резисторов
  1. Рассмотрим следующую схему, в которой четыре резистора R1, R2, R3 и R4 подключены параллельно.

Значения сопротивления каждого резистора:

R1 = 10 Ом

R2 = 20 Ом

R3 = 30 Ом

R4 = 40 Ом

Напряжение питания V = 24 В

Полный ток в схеме можно рассчитать двумя способами.

Первый метод — вычислить отдельные токи, протекающие через каждый резистор.

Если I1 — это ток, протекающий через резистор R1, то согласно закону Ома

I1 = V / R 1 = 24/10 = 2,4 A

Аналогично, если I2 — это ток, протекающий через резистор R2, то согласно по закону Ома

I2 = V / R 2 = 24/20 = 1,2 A

Если I3 — это ток, протекающий через резистор R3, то по закону Ома

I3 = V / R 3 = 24 / 30 = 0,8 А

А если I4 — ток, протекающий через резистор R4, то по закону Ома

I4 = V / R 4 = 24/40 = 0.6 A

Если I ИТОГО — это полный ток в цепи, то согласно закону Кирхгофа,

I ИТОГО = I1 + I2 + I3 + I4 = 2,4 + 1,2 + 0,8 + 0,6 = 5A

Второй метод расчета тока — это определение эквивалентного сопротивления цепи.

Эквивалентное сопротивление цепи составляет

1 / R EQ = (1 / R 1 ) + (1 / R 2 ) + (1 / R 3 ) + (1 / R 4 )

1 / R EQ = (1/10) + (1/20) + (1/30) + (1/40)

R EQ = 1/2.083 = 4,8 Ом

Этот единственный резистор можно использовать для замены всех резисторов в параллельной комбинации.

∴ I ИТОГО = V / R EQ = 24 / 4.8 = 5A.

Рассмотрим следующую схему, в которой три резистора R1, R2 и R3 соединены параллельно.

Ток, протекающий через R1, равен I1 = 6A

Ток, протекающий через R2, равен I2 = 4A

Ток, протекающий через R3, равен I3 = 2A

В параллельных резистивных цепях напряжение на каждом резисторе одинаково и равно напряжению питания.

Здесь напряжение питания V = 12 В.

Если V1 — это напряжение на резисторе R1, V2 — это напряжение на резисторе R2, а V3 — это напряжение на резисторе R3, тогда

V = V1 = V2 = V3 = 12 В

Тогда по закону Ома

R1 = V 1 / I 1

R1 = 12/6

R1 = 2 Ом

R2 = V 2 / I 2

R2 = 12/4

R2 = 3 Ом

R3 = V 3 / I 3

R3 = 12/2

R3 = 6 Ом

Применения

Концепция резисторов в parallel используется при анализе мостовой схемы Уитстона.Параллельно соединенные резисторы действуют как цепь делителя тока. Эта текущая концепция делителя используется в таких приложениях, как аналого-цифровые преобразователи и цифро-аналоговые преобразователи.

Как рассчитать падение напряжения на резисторе в параллельной цепи

Обновлено 28 декабря 2020 г.

Автор: S. Hussain Ather

••• Syed Hussain Ather

TL; DR (Too Long; Didn ‘ t Считать)

На приведенной выше схеме параллельной цепи падение напряжения можно найти, суммируя сопротивления каждого резистора и определяя, какое напряжение получается из тока в этой конфигурации.Эти примеры параллельных цепей иллюстрируют концепции тока и напряжения в разных ветвях.

На схеме параллельной цепи, падение напряжения на резисторе в параллельной цепи одинаково для всех резисторов в каждой ветви параллельной цепи. Напряжение, выраженное в вольтах, измеряет электродвижущую силу или разность потенциалов в цепи.

Если у вас есть цепь с известной величиной тока , поток электрического заряда, вы можете рассчитать падение напряжения в схемах параллельной цепи с помощью:

  1. Определите объединенное сопротивление или сопротивление к потоку заряда параллельных резисторов.Суммируйте их как 1 / R всего = 1 / R 1 + 1 / R 2 … для каждого резистора. Для приведенной выше параллельной цепи полное сопротивление можно найти как:
    1. 1 / R всего = 1/5 Ом + 1/6 Ом + 1/10 Ом
    2. 1 / R всего = 6/30 Ом + 5/30 Ом + 3/30 Ом
    3. 1 / R всего = 14/30 Ом
    4. R всего = 30/14 Ом = 15/7 Ом
  2. Умножьте ток на полное сопротивление, чтобы получить падение напряжения в соответствии с законом Ома В = IR .Это равно падению напряжения во всей параллельной цепи и на каждом резисторе в параллельной цепи. В этом примере падение напряжения равно В = 5 А x 15/7 Ом = 75/7 В.

Этот метод решения уравнений работает, потому что ток, входящий в любую точку параллельной цепи, должен быть равен текущий уход. Это происходит из-за действующего закона Кирхгофа , который гласит: «алгебраическая сумма токов в сети проводников, встречающихся в одной точке, равна нулю.»Калькулятор параллельной цепи мог бы использовать этот закон в ветвях параллельной цепи.

Если мы сравним ток, входящий в три ветви параллельной цепи, он должен равняться общему току, выходящему из ветвей. Поскольку падение напряжения остается постоянная на каждом параллельном резисторе, это падение напряжения, вы можете суммировать сопротивление каждого резистора, чтобы получить общее сопротивление и определить напряжение на основе этого значения. Примеры параллельных цепей показывают это

Падение напряжения в последовательной цепи

•• • Syed Hussain Ather

С другой стороны, в последовательной цепи вы можете рассчитать падение напряжения на каждом резисторе, зная, что в последовательной цепи ток постоянен на всем протяжении.Это означает, что падение напряжения на каждом резисторе разное и зависит от сопротивления в соответствии с законом Ома В = IR . В приведенном выше примере падение напряжения на каждом резисторе составляет:

V_1 = R_1I = 3 \ times 3 = 9 \ text {V} \\ V_2 = R_2I = 10 \ times 3 = 30 \ text {V} \\ V_3 = R_3I = 5 \ times 3 = 15 \ text {V}

Сумма каждого падения напряжения должна быть равна напряжению батареи в последовательной цепи. Это означает, что наша батарея имеет напряжение 54 В.

Этот метод решения уравнений работает, потому что падение напряжения на всех резисторах, расположенных последовательно, должно в сумме составлять общее напряжение последовательной цепи.Это происходит из-за закона напряжения Кирхгофа , который гласит, что «направленная сумма разностей потенциалов (напряжений) вокруг любого замкнутого контура равна нулю». Это означает, что в любой точке замкнутой последовательной цепи падение напряжения на каждом резисторе должно равняться общему напряжению цепи. Поскольку ток в последовательной цепи постоянный, падение напряжения должно различаться для каждого резистора.

Параллельные и последовательные схемы

В параллельной схеме все компоненты схемы подключаются между одними и теми же точками на схеме.Это дает им их разветвленную структуру, в которой ток разделяется между каждой ветвью, но падение напряжения на каждой ветви остается неизменным. Сумма каждого резистора дает общее сопротивление на основе обратной величины каждого сопротивления ( 1 / R всего = 1 / R 1 + 1 / R 2 для каждого резистора).

Напротив, в последовательной цепи есть только один путь для прохождения тока. Это означает, что ток остается постоянным на всем протяжении, а падение напряжения на каждом резисторе отличается.Сумма каждого резистора дает общее сопротивление при линейном суммировании ( R всего = R 1 + R 2 для каждого резистора).

Последовательно-параллельные схемы

Вы можете использовать оба закона Кирхгофа для любой точки или петли в любой цепи и применять их для определения напряжения и тока. Законы Кирхгофа дают вам метод определения тока и напряжения в ситуациях, когда природа цепи как последовательной и параллельной может быть не такой простой.

Как правило, для схем, в которых есть как последовательные, так и параллельные компоненты, вы можете рассматривать отдельные части схемы как последовательные или параллельные и соответственно комбинировать их.

Эти сложные последовательно-параллельные схемы можно решить несколькими способами. Один из методов — рассматривать их части как параллельные или последовательные. Другой метод — использование законов Кирхгофа для определения обобщенных решений, использующих систему уравнений. Калькулятор последовательно-параллельных цепей учитывает различную природу цепей.

••• Syed Hussain Ather

В приведенном выше примере текущая точка выхода A должна равняться текущей точке выхода A. Это означает, что вы можете написать:

(1). I_1 = I_2 + I_3 \ text {или} I_1-I_2-I_3 = 0

Если рассматривать верхний контур как замкнутую последовательную цепь и рассматривать падение напряжения на каждом резисторе с использованием закона Ома с соответствующим сопротивлением, вы можете написать:

(2). V_1-R_1I_1-R_2I_2 = 0

и, проделав то же самое для нижнего контура, вы можете обрабатывать каждое падение напряжения в направлении тока в зависимости от тока и сопротивления, чтобы записать:

(3).V_1 + V_2 + R_3I_3-R_2I_2 = 0

Это дает вам три уравнения, которые можно решить несколькими способами. Вы можете переписать каждое из уравнений (1) — (3) так, чтобы напряжение было с одной стороны, а ток и сопротивление — с другой. Таким образом, вы можете рассматривать три уравнения как зависимые от трех переменных I 1 , I 2 и I 3 с коэффициентами комбинаций 1 , 2 и 3 .

\ начало {выровнено} & (1).I_1-I_2-I_3 = 0 \\ & (2). R_1I_1 + R_2I_2 + 0 \ times I_3 = V_1 \\ & (3). 0 \ times I_1 + R_2I_2-R_3I_3 = V_1 + V_2 \ end {выровнено}

Эти три уравнения демонстрируют, как напряжение в каждой точке цепи каким-то образом зависит от тока и сопротивления. Если вы помните законы Кирхгофа, вы можете создать эти обобщенные решения схемных задач и использовать матричную нотацию для их решения. Таким образом, вы можете подставить значения для двух величин (среди которых напряжение, ток, сопротивление), чтобы найти третью.

19.3 Параллельные схемы — Физика

Задачи обучения разделу

К концу этого раздела вы сможете делать следующее:

  • Расшифровка принципиальных схем с параллельными резисторами
  • Вычислить эквивалентное сопротивление комбинаций резисторов, содержащих последовательные и параллельные резисторы

Поддержка учителей

Поддержка учителей

Цели обучения в этом разделе помогут вашим ученикам овладеть следующими стандартами:

  • (5) Научные концепции.Студент знает природу сил в физическом мире. Ожидается, что студент:
    • (ж) дизайн. построить и рассчитать в терминах сквозного тока, разности потенциалов, сопротивления и мощности, используемой элементами электрической цепи, соединенными как в последовательной, так и в параллельной комбинациях.

Кроме того, в Руководстве по лаборатории физики средней школы рассматривается содержание этого раздела лаборатории под названием «Схемы», а также следующие стандарты:

  • (5) Учащийся знает природу сил в физическом мире.Ожидается, что студент:
    • (F) проектировать, конструировать и рассчитывать в терминах сквозного тока, разности потенциалов, сопротивления и мощности, используемой элементами электрической цепи, соединенными как в последовательной, так и в параллельной комбинациях.

Раздел Основные термины

Параллельные резисторы

В предыдущем разделе мы узнали, что последовательно включенные резисторы — это резисторы, которые подключаются друг за другом. Если вместо этого мы объединим резисторы, подключив их рядом друг с другом, как показано на рисунке 19.16, то говорят, что резисторы подключены параллельно . Резисторы включены параллельно, когда оба конца каждого резистора соединены непосредственно вместе.

Обратите внимание, что верхние части резисторов подключены к одному и тому же проводу, поэтому напряжение на верхушках каждого резистора одинаково. Аналогичным образом, нижние части резисторов подключены к одному и тому же проводу, поэтому напряжение на нижней стороне каждого резистора одинаково. Это означает, что падение напряжения на каждом резисторе одинаковое.В этом случае падение напряжения соответствует номинальному напряжению В батареи, потому что верхний и нижний провода подключаются к положительной и отрицательной клеммам батареи соответственно.

Хотя падение напряжения на каждом резисторе одинаково, мы не можем сказать то же самое для тока, протекающего через каждый резистор. Таким образом, I1, I2 и I3I1, I2 и I3 не обязательно одинаковы, потому что резисторы R1, R2 и R3R1, R2 и R3 не обязательно имеют одинаковое сопротивление.

Обратите внимание, что три резистора на рисунке 19.16 обеспечивают три разных пути, по которым может течь ток. Это означает, что эквивалентное сопротивление для этих трех резисторов должно быть меньше наименьшего из трех резисторов. Чтобы понять это, представьте, что наименьший резистор — это единственный путь, по которому может течь ток. Теперь добавьте альтернативные пути, подключив другие резисторы параллельно. Поскольку у тока больше путей, общее сопротивление (т. Е. Эквивалентное сопротивление) будет уменьшаться. Следовательно, эквивалентное сопротивление должно быть меньше наименьшего сопротивления параллельных резисторов.

Рисунок 19.16 На левой принципиальной схеме показаны три резистора, включенных параллельно. Напряжение В батареи приложено ко всем трем резисторам. Токи, протекающие через каждую ветвь, не обязательно равны. На правой принципиальной схеме показано эквивалентное сопротивление, заменяющее три параллельных резистора.

Teacher Support

Teacher Support

Подчеркните, что напряжение на каждом параллельном резисторе одинаковое, а ток может отличаться; то же самое будет, если пара резисторов будет иметь одинаковое сопротивление.

Чтобы найти эквивалентное сопротивление RequivRequiv трех резисторов R1, R2 и R3R1, R2 и R3, мы применим закон Ома к каждому резистору. Поскольку падение напряжения на каждом резисторе составляет В , получаем

V = I1R1, V = I2R2, V = I3R3V = I1R1, V = I2R2, V = I3R3

19,21

или

I1 = VR1, I2 = VR2, I3 = VR3. I1 = VR1, I2 = VR2, I3 = VR3.

19,22

Из сохранения заряда мы также знаем, что три тока I1, I2 и I3I1, I2 и I3 должны складываться, чтобы получить ток I , который проходит через батарею.Если бы это было не так, ток должен был бы таинственным образом создаваться или разрушаться где-то в цепи, что физически невозможно. Таким образом, имеем

Я = I1 + I2 + I3. I = I1 + I2 + I3.

19,23

Вставка выражений для I1, I2 и I3I1, I2 и I3 в это уравнение дает

I = VR1 + VR2 + VR3 = V (1R1 + 1R2 + 1R3) I = VR1 + VR2 + VR3 = V (1R1 + 1R2 + 1R3)

19,24

или

V = I (11 / R1 + 1 / R2 + 1 / R3). V = I (11 / R1 + 1 / R2 + 1 / R3).

19,25

Эта формула представляет собой закон Ома, где множитель в скобках представляет собой эквивалентное сопротивление.

V = I (11 / R1 + 1 / R2 + 1 / R3) = IRэкв. V = I (11 / R1 + 1 / R2 + 1 / R3) = IRequiv.

19,26

Таким образом, эквивалентное сопротивление для трех параллельно включенных резисторов составляет

Требование = 11 / R1 + 1 / R2 + 1 / R3. Требование = 11 / R1 + 1 / R2 + 1 / R3.

19,27

Та же самая логика работает для любого количества резисторов, включенных параллельно, поэтому общая форма уравнения, которая дает эквивалентное сопротивление резисторов N , подключенных параллельно, составляет

Требование = 11 / R1 + 1 / R2 + ⋯ + 1 / RN. Требование = 11 / R1 + 1 / R2 + ⋯ + 1 / RN.

19,28

Рабочий пример

Найдите ток через параллельные резисторы

Три схемы ниже эквивалентны.Если номинальное напряжение батареи Vbattery = 3VVbattery = 3V, каково эквивалентное сопротивление цепи и какой ток проходит через цепь?

Стратегия

Три резистора подключены параллельно, и падение напряжения на них составляет В аккумулятор . Таким образом, мы можем применить уравнение для эквивалентного сопротивления резисторов, включенных параллельно, которое принимает вид

Требование = 11 / R1 + 1 / R2 + 1 / R3. Требование = 11 / R1 + 1 / R2 + 1 / R3.

19,29

Схема с эквивалентным сопротивлением показана ниже.Как только мы узнаем эквивалентное сопротивление, мы можем использовать закон Ома, чтобы найти ток в цепи.

Решение

Вставка данных значений сопротивления в уравнение эквивалентного сопротивления дает

Требуемое = 11 / R1 + 1 / R2 + 1 / R3 = 11/10 Ом + 1/25 Ом + 1/15 Ом = 4,84 Ом Требуемое = 11 / R1 + 1 / R2 + 1 / R3 = 11/10 Ом + 1/25 Ом + 1/15 Ом = 4,84 Ом.

19,30

Таким образом, ток в цепи равен

V = IRI = VR = 3 В 4,84 Ом = 0,62 А. V = IRI = VR = 3 В 4,84 Ом = 0,62 А.

19,31

Обсуждение

Хотя 0.62 А протекает через всю цепь, обратите внимание, что этот ток не проходит через каждый резистор. Однако, поскольку электрический заряд должен сохраняться в цепи, сумма токов, проходящих через каждую ветвь цепи, должна составлять ток, проходящий через батарею. Другими словами, мы не можем волшебным образом создать заряд где-нибудь в цепи и добавить этот новый заряд к току. Давайте проверим это рассуждение, используя закон Ома, чтобы найти ток через каждый резистор.

I1 = VR1 = 3 В 10 Ом = 0.30AI2 = VR2 = 3V25Ω = 0.12AI3 = VR3 = 3V15Ω = 0.20AI1 = VR1 = 3V10Ω = 0.30AI2 = VR2 = 3V25Ω = 0.12AI3 = VR3 = 3V15Ω = 0.20A

19,32

Как и ожидалось, эти токи в сумме дают 0,62 A, который представляет собой обнаруженный полный ток, проходящий через эквивалентный резистор. Также обратите внимание, что наименьший резистор имеет наибольший ток, протекающий через него, и наоборот.

Рабочий пример

Рассуждения с параллельными резисторами

Без каких-либо расчетов, каково эквивалентное сопротивление трех одинаковых резисторов R , включенных параллельно?

Стратегия

Три идентичных резистора R , включенных параллельно, образуют три идентичных пути, по которым может течь ток.Таким образом, току протекать через эти резисторы в три раза легче, чем через один из них.

Решение

Если протекать через три одинаковых резистора R в три раза легче, чем через один из них, эквивалентное сопротивление должно быть втрое меньше: R /3.

Обсуждение

Давайте проверим наши рассуждения, вычислив эквивалентное сопротивление трех одинаковых резисторов R , включенных параллельно.Уравнение эквивалентного сопротивления параллельно включенных резисторов дает

Требуется = 11 / R + 1 / R + 1 / R = 13 / R = R3. Требуется = 11 / R + 1 / R + 1 / R = 13 / R = R3.

19,33

Таким образом, наши рассуждения были правильными. В общем, когда доступно больше путей, по которым может течь ток, эквивалентное сопротивление уменьшается. Например, если у нас есть идентичные резисторы R , подключенные параллельно, эквивалентное сопротивление будет R /10.

Практические задачи

10.

Три резистора 10, 20 и 30 Ом подключены параллельно.Какое эквивалентное сопротивление?

  1. Эквивалентное сопротивление 5,5 Ом
  2. Эквивалентное сопротивление 60 Ом
  3. Эквивалентное сопротивление 6 × 103 Ом
  4. Эквивалентное сопротивление составляет 6 × 104 Ом
11.

Если падение напряжения на 5 \ text {-V} происходит на R_1, а R_1 подключен параллельно к R_2, каково падение напряжения на R_2?

  1. Падение напряжения на 0 \, \ text {V}.
  2. Падение напряжения на 2.5 \, \ text {V}.
  3. Падение напряжения составляет 5 \, \ text {V}.
  4. Падение напряжения составляет 10 \, \ text {V}.

Резисторы параллельно и последовательно

Более сложные соединения резисторов иногда представляют собой просто комбинации последовательного и параллельного. Комбинации последовательных и параллельных резисторов могут быть уменьшены до одного эквивалентного сопротивления с помощью техники, показанной на рисунке 19.17. Различные части идентифицируются как последовательные или параллельные, уменьшаются до их эквивалентов и далее уменьшаются до тех пор, пока не останется единственное сопротивление.Процесс занимает больше времени, чем труден.

Рис. 19.17 Эта комбинация из семи резисторов имеет как последовательные, так и параллельные части. Каждый из них идентифицируется и приводится к эквивалентному сопротивлению, а затем уменьшается до тех пор, пока не будет достигнуто одно эквивалентное сопротивление.

Поддержка учителей

Поддержка учителей
Предупреждение о заблуждении

У студентов может возникнуть соблазн немедленно сложить R1R1 и R7R7 вместе, потому что они кажутся последовательными.Обратите внимание, что R1R1 включен последовательно с параллельной комбинацией R7R7 и всех резисторов справа от R7R7. Таким образом, перед добавлением к R1R1 необходимо найти эквивалентное сопротивление этой параллельной комбинации.

Поддержка учителей

Поддержка учителей

Рассмотрите этот пример вместе с учащимися, чтобы убедиться, что они понимают сокращение, которое происходит на каждом этапе.

Давайте проработаем четыре шага на рисунке 19.17, чтобы уменьшить семь резисторов до одного эквивалентного резистора.Чтобы не отвлекать внимание от алгебры, предположим, что каждый резистор имеет сопротивление 10 Ом. На шаге 1 мы уменьшаем два набора параллельных резисторов, обведенных синей пунктирной петлей. Верхний набор имеет три резистора, включенных параллельно, и будет уменьшен до одного эквивалентного резистора RP1RP1. Нижний набор имеет два резистора, включенных параллельно, и будет уменьшен до одного эквивалентного резистора RP2RP2. Используя уравнение эквивалентного сопротивления параллельно включенных резисторов, получаем

RP1 = 11 / R2 + 1 / R3 + 1 / R4 = 11 / 10Ω + 1 / 10Ω + 1 / 10Ω = 103Ω RP2 = 11 / R5 + 1 / R6 = 11 / 10Ω + 1 / 10Ω = 5Ω.RP1 = 11 / R2 + 1 / R3 + 1 / R4 = 11 / 10Ω + 1 / 10Ω + 1 / 10Ω = 103Ω RP2 = 11 / R5 + 1 / R6 = 11 / 10Ω + 1 / 10Ω = 5Ω.

19,34

Эти два эквивалентных сопротивления обведены красной пунктирной петлей после шага 1. Они включены последовательно, поэтому мы можем использовать уравнение для эквивалентного сопротивления последовательно включенных резисторов, чтобы уменьшить их до одного эквивалентного сопротивления RS1RS1. Это делается на шаге 2, в результате получается

. RS1 = RP1 + RP2 = 103 Ом + 5 Ом = 253 Ом. RS1 = RP1 + RP2 = 103 Ом + 5 Ом = 253 Ом.

19,35

Эквивалентный резистор RS1RS1 появляется в зеленой пунктирной петле после шага 2.Этот резистор включен параллельно резистору R7R7, поэтому пара может быть заменена эквивалентным резистором RP3RP3, который равен

. RP3 = 11 / RS1 + 1 / R7 = 13 / 25Ω + 1 / 10Ω = 5011Ω. RP3 = 11 / RS1 + 1 / R7 = 13 / 25Ω + 1 / 10Ω = 5011Ω.

19,36

Это делается на шаге 3. Резистор RP3RP3 включен последовательно с резистором R1R1, как показано в фиолетовой пунктирной петле после шага 3. Эти два резистора объединяются на последнем шаге, чтобы сформировать окончательный эквивалент резистора RequivRequiv, что

Requiv = R1 + RP3 = 10 Ом + 5011 Ом = 16011 Ом.Requiv = R1 + RP3 = 10 Ом + 5011 Ом = 16011 Ом.

19,37

Таким образом, вся комбинация из семи резисторов может быть заменена одним резистором с сопротивлением около 14,5 Ом.

Это была большая работа, и вы можете спросить, зачем мы ее делаем. Для нас важно знать эквивалентное сопротивление всей цепи, чтобы мы могли рассчитать ток, протекающий по цепи. Закон Ома говорит нам, что ток, протекающий по цепи, зависит от сопротивления цепи и напряжения в цепи.Но чтобы узнать силу тока, мы должны сначала узнать эквивалентное сопротивление.

Вот общий подход к поиску эквивалентного резистора для любой произвольной комбинации резисторов:

  1. Определите группу резисторов, которые включены только параллельно или только последовательно.
  2. Для резисторов, подключенных последовательно, используйте уравнение для эквивалентного сопротивления резисторов, подключенных последовательно, чтобы уменьшить их до одного эквивалентного сопротивления. Для резисторов, подключенных параллельно, используйте уравнение для эквивалентного сопротивления резисторов, подключенных параллельно, чтобы уменьшить их до одного эквивалентного сопротивления.
  3. Нарисуйте новую принципиальную схему с резисторами из шага 1, замененными их эквивалентными резисторами.
  4. Если в цепи осталось более одного резистора, вернитесь к шагу 1 и повторите. В противном случае все готово.

Развлечение по физике

Робот

Роботы захватывают наше коллективное воображение уже более века. Теперь мечта о создании умных машин для выполнения нашей грязной работы, а иногда и просто для того, чтобы составить нам компанию, становится реальностью. Робототехника стала огромной областью исследований и разработок, причем некоторые технологии уже коммерциализированы.Подумайте, например, о небольших автономных пылесосах.

На рис. 19.18 показаны лишь некоторые из множества различных форм, которые могут принимать роботы. Самые продвинутые роботы-гуманоиды могут ходить, наливать напитки и даже танцевать (хотя и не очень изящно). Другие роботы вдохновлены биологией, например, собачий робот , показанный на средней фотографии рис. 19.18. Этот робот может нести сотни фунтов груза по пересеченной местности. Фотография справа на рис. 19.18 показывает внутреннюю работу блока M, , разработанного Массачусетским технологическим институтом.Эти простые на вид блоки содержат инерционные колеса и электромагниты, которые позволяют им вращаться, переворачиваться в воздух и соединяться друг с другом в самых разных формах. Обмениваясь беспроводной связью между собой, они самостоятельно собираются в различные формы, такие как столы, стулья и, возможно, когда-нибудь даже здания.

Все роботы включают в себя огромное количество физики и инженерии. Простое наливание напитка было освоено роботами совсем недавно, после более 30 лет исследований и разработок! Баланс и выбор времени, которые мы, люди, считаем само собой разумеющимися, на самом деле являются очень сложной задачей, требующей отличного баланса, ловкости и обратной связи.Чтобы справиться с этим, требуются датчики для обнаружения баланса, вычислительная мощность для анализа данных и передачи соответствующих компенсирующих действий, а также соединения и приводы для выполнения требуемых действий.

Помимо определения силы тяжести или ускорения, роботы могут содержать несколько различных датчиков для обнаружения света, звука, температуры, запаха, вкуса и т. Д. Все эти устройства основаны на физических принципах, которые вы изучаете в этом тексте. Например, оптика, используемая для машинного зрения, аналогична оптике, используемой в ваших цифровых камерах: пиксельные полупроводниковые детекторы, в которых свет преобразуется в электрические сигналы.Для определения температуры можно использовать простые термисторы, которые представляют собой резисторы, сопротивление которых изменяется в зависимости от температуры.

Сегодня построить робота гораздо проще, чем несколько лет назад. Многие компании сейчас предлагают комплекты для сборки роботов. Они варьируются по сложности, от чего-то подходящего для младших школьников до чего-то, что бросает вызов лучшим профессиональным инженерам. Если интересно, вы можете легко найти их в Интернете и начать создавать своего собственного робота уже сегодня.

Рис. 19.18 Роботы бывают разных форм и размеров, от классического гуманоида типа до собачьих роботов до маленьких кубиков, которые самостоятельно собираются для выполнения разнообразных задач.

Watch Physics

Параллельные резисторы

На этом видео лектор обсуждает простую схему с батареей и парой резисторов, включенных параллельно. Он подчеркивает, что электроны текут в направлении, противоположном положительному току, а также использует тот факт, что напряжение одинаково во всех точках идеального провода.Вывод очень похож на то, что делается в этом тексте, но лектор хорошо его проходит, объясняя каждый шаг.

Захват

Верно или нет. На принципиальной схеме мы можем предположить, что напряжение одинаково в каждой точке данного провода.

  1. ложный
  2. правда

Watch Physics

Последовательные и параллельные резисторы

В этом видео показано, как рассчитать эквивалентное сопротивление цепи, содержащей резисторы, включенные параллельно и последовательно.Лектор использует тот же подход, что и описанный выше, для поиска эквивалентного сопротивления.

Захват

Представьте, что параллельно подключены N одинаковых резисторов. Каждый резистор имеет сопротивление R . Какое эквивалентное сопротивление для этой группы параллельных резисторов?

  1. Эквивалентное сопротивление ( R ) N .
  2. Эквивалентное сопротивление — NR.
  3. Эквивалентное сопротивление RN.РН.
  4. Эквивалентное сопротивление — NR.NR.

Рабочий пример

Найти ток через схему комплексного резистора

Батарея в цепи ниже имеет номинальное напряжение 10 В. Какой ток течет по цепи и в каком направлении?

Стратегия

Примените стратегию поиска эквивалентного сопротивления, чтобы заменить все резисторы одним эквивалентным сопротивлением, затем используйте закон Ома, чтобы найти ток через эквивалентный резистор.

Решение

Комбинацию резисторов R4R4 и R5R5 можно уменьшить до эквивалентного сопротивления

RP1 = 11 / R4 + 1 / R5 = 11/45 Ом + 1/60 Ом = 25,71 Ом R. RP1 = 11 / R4 + 1 / R5 = 11/45 Ом + 1/60 Ом = 25,71 Ом R.

19,38

Замена R4R4 и R5R5 с этим эквивалентным сопротивлением дает схему ниже.

Теперь мы заменим два верхних резистора R2R2 и R3R3 эквивалентным резистором RS1RS1 и два нижних резистора RP1RP1 и R6R6 их эквивалентным резистором RS2RS2.Эти резисторы включены последовательно, поэтому мы складываем их вместе, чтобы найти эквивалентное сопротивление.

RS1 = R2 + R3 = 50 Ом + 30 Ом = 80 Ом RS2 = RP1 + R6 = 25,71 Ом + 20 Ом = 45,71 Ом RS1 = R2 + R3 = 50 Ом + 30 Ом = 80 Ом RS2 = RP1 + R6 = 25,71 Ом + 20 Ом = 45,71 Ом

19,39

Замена соответствующих резисторов на их эквивалентные резисторы дает схему, приведенную ниже.

Теперь замените два резистора RS1 и RS2RS1 и RS2, которые включены параллельно, на их эквивалентные резисторы RP2RP2. Сопротивление RP2RP2

RP2 = 11 / RS1 + 1 / RS2 = 11/80 Ом + 1/45.71 Ом = 29,09 Ом. RP2 = 11 / RS1 + 1 / RS2 = 11/80 Ом + 1 / 45,71 Ом = 29,09 Ом.

19,40

Обновление принципиальной схемы путем замены RS1 и RS2 RS1 и RS2 на это эквивалентное сопротивление дает схему ниже.

Наконец, мы объединяем резисторы R1 и RP2R1 и RP2, которые включены последовательно. Эквивалентное сопротивление: RS3 = R1 + RP2 = 75 Ом + 29,09 Ом = 104,09 Ом. RS3 = R1 + RP2 = 75 Ом + 29,09 Ом = 104,09 Ом. Окончательная схема показана ниже.

Теперь мы используем закон Ома, чтобы найти ток в цепи.

V = IRS3I = VRS3 = 10V104.09Ω = 0,096AV = IRS3I = VRS3 = 10V 104,09Ω = 0,096A

19,41

Ток идет от положительной клеммы батареи к отрицательной клемме батареи, поэтому в этой цепи он течет по часовой стрелке.

Обсуждение

Этот расчет может показаться довольно длинным, но, немного попрактиковавшись, вы сможете объединить несколько этапов. Также обратите внимание, что при вычислении учитывались лишние значащие цифры. Только в конце окончательный результат был округлен до двух значащих цифр.

Рабочий пример

Странные схемы цепей

Иногда вы можете встретить принципиальные схемы, которые нарисованы не очень аккуратно, например, схему, показанную ниже.Эта принципиальная схема больше похожа на то, как настоящая схема может появиться на лабораторном столе. Каково эквивалентное сопротивление резисторов на этой диаграмме, если каждый резистор имеет сопротивление 10 Ом и номинальное напряжение батареи 12 В.

Стратегия

Давайте перерисуем эту принципиальную схему, чтобы было понятнее. Затем мы применим описанную выше стратегию для расчета эквивалентного сопротивления.

Решение

Чтобы перерисовать диаграмму, рассмотрите рисунок ниже.В верхней схеме синие резисторы образуют путь от положительной клеммы батареи к отрицательной. Параллельно с этой цепью расположены красные резисторы, которые составляют еще один путь от положительной к отрицательной клемме батареи. Синий и красный пути показаны более четко на нижней принципиальной схеме. Обратите внимание, что как на верхней, так и на нижней принципиальной схеме синий и красный пути соединяют положительную клемму аккумулятора с отрицательной клеммой аккумулятора.

Теперь легче увидеть, что R1 и R2R1 и R2 подключены параллельно, а параллельная комбинация находится последовательно с R4R4. Эта комбинация, в свою очередь, параллельна последовательной комбинации R3 и R5R3 и R5. Сначала мы вычисляем синюю ветвь, которая содержит R1, R2 и R4R1, R2 и R4. Эквивалентное сопротивление

Rblue = 11 / R1 + 1 / R2 + R4 = 11 / 10Ω + 1 / 10Ω + 10Ω = 15Ω. Rblue = 11 / R1 + 1 / R2 + R4 = 11 / 10Ω + 1 / 10Ω + 10Ω = 15Ω.

19,42

где мы показываем вклад от параллельной комбинации резисторов и от последовательной комбинации резисторов.Теперь рассчитаем эквивалентное сопротивление красной ветви, которое составляет

. Rred = R3 + R5 = 10 Ом + 10 Ом = 20 Ом. Rred = R3 + R5 = 10 Ом + 10 Ом = 20 Ом.

19,43

Если вставить эти эквивалентные резисторы в схему, получится схема, показанная ниже.

Эти два резистора включены параллельно, поэтому их можно заменить одним эквивалентным резистором с сопротивлением

Requiv = 11 / Rblue + 1 / Rred = 11 / 15Ω + 1 / 20Ω = 8,6Ω. Requiv = 11 / Rblue + 1 / Rred = 11 / 15Ω + 1 / 20Ω = 8.6Ω.

19,44

Окончательная эквивалентная схема показана ниже.

Обсуждение

Найти эквивалентное сопротивление было проще с помощью четкой принципиальной схемы. Вот почему мы стараемся делать четкие принципиальные схемы, где резисторы, включенные параллельно, выстроены параллельно друг другу и в одном и том же горизонтальном положении на схеме.

Теперь мы можем использовать закон Ома, чтобы найти ток, проходящий через каждую ветвь этой цепи. Рассмотрим принципиальную схему с RblueRblue и RredRred. Напряжение на каждой из этих ветвей составляет 12 В (т.е.е. номинальное напряжение аккумулятора). Ток в синей ветке —

Iblue = VRblue = 12В15Ω = 0,80A Синий = VRblue = 12V15Ω = 0,80A.

19,45

Ток через красную ветвь

Ired = VRred = 12 В20 Ом = 0,60 А. Ired = VRred = 12 В 20 Ом = 0,60 А.

19,46

Ток, протекающий через батарею, должен быть суммой этих двух токов (вы понимаете, почему?), Или 1,4 А.

Практические задачи

12.

Какова формула эквивалентного сопротивления двух параллельных резисторов с сопротивлением R 1 и R 2 ?

  1. Эквивалентное сопротивление двух параллельных резисторов Reqv = R1 + R2 Reqv = R1 + R2
  2. Эквивалентное сопротивление двух параллельных резисторов Reqv = R1 × R2Reqv = R1 × R2
  3. Эквивалентное сопротивление двух параллельных резисторов Reqv = R1-R2Reqv = R1-R2
  4. Эквивалентное сопротивление двух параллельных резисторов Reqv = 11 / R1 + 1 / R2Reqv = 11 / R1 + 1 / R2
13.

Рисунок 19.19

Какое эквивалентное сопротивление для двух показанных резисторов?

  1. Эквивалентное сопротивление 20 Ом
  2. Эквивалентное сопротивление 21 Ом
  3. Эквивалентное сопротивление 90 Ом
  4. Эквивалентное сопротивление 1,925 Ом

Проверьте свое понимание

14.

Падение напряжения на параллельных резисторах ________.

  1. то же для всех резисторов
  2. больше для больших резисторов
  3. На
  4. меньше для больших резисторов
  5. больше для меньших резисторов
15.

Рассмотрим схему из параллельных резисторов. Наименьший резистор — 25 Ом. Каков верхний предел эквивалентного сопротивления?

  1. Верхний предел эквивалентного сопротивления составляет 2,5 Ом.
  2. Верхний предел эквивалентного сопротивления составляет 25 Ом.
  3. Верхний предел эквивалентного сопротивления составляет 100 Ом.
  4. Нет верхнего предела.
Сопротивления

параллельно, онлайн калькулятор и формулы


Калькулятор и формулы для расчета резисторов параллельно

Рассчитать сопротивление параллельного соединения

При параллельном подключении резисторов ток распределяется по отдельным резисторам.

Рассчитать общее сопротивление

Экспоненты не допускаются. Введите все значения в подходящих равных единицах измерения. Если вы введете все значения, например в кОм результат также отображается в кОм.

Для расчета введите значения отдельных резисторов, разделенных точкой с запятой.
Пример: 33; 12,1; 22


Сопротивление параллельно вычислителю


Формулы резисторов, включенных параллельно

Чтобы рассчитать общее сопротивление нескольких резисторов, включенных параллельно, их значения проводимости складываются.Проводимость обратно пропорциональна сопротивлению. Формула для трех резисторов, соединенных параллельно:

\ (\ displaystyle \ frac {1} {R_ {ges}} = \ frac {1} {R_1} + \ frac {1} {R_2} + \ frac {1} {R_3} \)

Если необходимо рассчитать общее сопротивление двух параллельных резисторов, можно использовать следующую формулу.

\ (\ Displaystyle R_ {ges} = \ гидроразрыва {R_1 · R_2} {R_1 + R_2} \)

Эта страница полезна? да Нет

Спасибо за ваш отзыв!

Прошу прощения за это

Как мы можем это улучшить?

послать


Электрическое сопротивление в последовательных и параллельных сетях

Последовательное соединение

Общее сопротивление для резисторов, подключенных последовательно, можно рассчитать как

R = R 1 + R 2 +…. + R n (1)

где

R = сопротивление (Ом, Ом)

Пример — Последовательные резисторы

Три резистора 33 Ом , 33 Ом и 47 Ом подключены последовательно. Общее сопротивление можно рассчитать как

R = ( 33 Ом) + ( 33 Ом) + ( 47 Ом)

= 113 Ом

Параллельное соединение

Общее сопротивление для резисторов, подключенных параллельно, можно рассчитать как

1 / R = 1 / R 1 + 1 / R 2 +…. + 1 / R n (2)

Эквивалентное сопротивление двух параллельно подключенных резисторов можно выразить как

R = R 1 R 2 / (R 1 + R 2 ) (3)

Пример — параллельные резисторы

Три резистора 33 Ом , 33 Ом и 47 Ом подключены параллельно. Общее сопротивление можно рассчитать как

1 / R = 1 / ( 33 Ом ) + 1 / ( 33 Ом ) + 1 / (47 Ом )

= 0.082 (1 / Ом)

R = 1 / (0,082 Ом)

= 12,2 Ом

Если напряжение батареи 12 В — ток в цепи можно рассчитать с помощью закон

I = U / R

= (12 В) / (12,2 Ом)

= 0,98 ампер

Можно рассчитать ток через каждый резистор

I 1 = U / R 1 = (12 В) / (33 Ом) = 0.36 ампер

I 2 = U / R 2 = (12 В) / (33 Ом) = 0,36 ампер

I 3 = U / R 3 = (12 В) / (47 Ом) = 0,26 ампера

Параллельно подключенные резисторы — Калькулятор

Сложите сопротивления до пяти параллельно подключенных резисторов и (необязательно) напряжение цепи.

Общее сопротивление и ток — а также отдельные токи во всех резисторах — будут рассчитаны:

R 1 (Ом)

R 2 (Ом)

R 3 ( Ом)

R 4 (Ом )

R 5 (Ом)

Напряжение (В)

I 1 (амперы)

I 9000 2 (амперы)

I 3 (амперы)

I 4 (амперы)

I 5 (амперы)

R (ом)

I (амперы)

.

Добавить комментарий

Ваш адрес email не будет опубликован. Обязательные поля помечены *